You are on page 1of 55

1.

“Accounting is a service activity”, definition was provided by:


a. Accounting Standards Council
b. Committee on Accounting Terminology of AICPA
c. American Accounting Association
d. International Accounting Standards Board

2. “The accounting function is to provide quantitative information, primarily financial in nature, about,
economic entities, that is intended to be useful in making economic decision”, definition was provided
by:
a. Accounting Standards Council
b. Committee on Accounting Terminology of AICPA
c. American Accounting Association
d. International Accounting Standards Board

3. “Accounting is the art of recording, classifying and summarizing in a significant manner and in terms
of money, transactions and events which are in part at least of a financial character and interpreting
the results thereof”, definition was provided by:
a. Accounting Standards Council
b. Committee on Accounting Terminology of AICPA
c. American Accounting Association
d. International Accounting Standards Board

4. “Accounting is the process of identifying, measuring and communicating economic information to


permit informed judgment and decision by users of the information”, definition was provided by:
a. Accounting Standards Council
b. Committee on Accounting Terminology of AICPA
c. American Accounting Association
d. International Accounting Standards Board

5. The accounting definition of American Accounting Association states that the very purpose of
accounting is to provide quantitative information to be useful in making an economic decision. Which
of the following components of it are false?
I. Identifying as the analytical component
II. Measuring as the technical component
III. Communicating as the formal component

a. I only c. I, II and III


b. I and II only d. none of the choices

6. The accounting process of recognition or nonrecognition of business activities as accountable events


is
a. Communicating c. Identifying
b. Measuring d. Recording

7. S1. All business activities are accountable.


S2. An event is unaccountable and unquantifiable when it does not affect assets, liabilities and equity.
a. True, True c. False, True
b. True, False d. False, False

8. S1. Economic activity comprises of economic resources and economic obligations, and are
emphasized and recognized in accounting.
S2. Sociological and psychological matters are accountable such as entering into a contract and death
of all the board of directors.
a. True, True c. False, True
b. True, False d. False, False
9. External transactions or exchange transactions are those economic events involving one entity and
another entity. Which of the following are external transactions?
I. Purchase of goods from a supplier
II. Borrowing money from a bank
III. Sale of goods to a customer
IV. Payment of salaries to employees
V. Payment of taxes to the government
VI. Production and casualty loss
a. I only c. I, II, III, IV and V only
b. I, II, III d. I to VI

10. The accounting process of assigning monetary amounts to the accountable economic transactions
and events is
a. Communicating c. Identifying
b. Measuring d. Recording

11. S1. Financial statements without monetary amounts would be largely unintelligible or
incomprehensible.
S2. US Dollar is the generally accepted unit of measuring accountable economic transactions
a. True, True c. False, True
b. True, False d. False, False

12. S1. Current value is the most common measure of financial transactions
S2. Historical cost includes fair value, value in use, fulfillment value and current cost.
a. True, True c. False, True
b. True, False d. False, False

13. The accounting process of preparing and distributing accounting reports to potential users of
accounting information is
a. Communicating c. Identifying
b. Measuring d. Recording

14. S1. Communicating includes recording, classifying and summarizing economic transactions and events
S2. Identifying and measuring are pointless if the information contained in the accounting records
cannot be communicated in some form to potential users.
a. True, True c. False, True
b. True, False d. False, False

15. The process of systematically maintaining a record of all economic business transactions after they
have need identified and measured is
a. Analyzing c. Classifying
b. Recording d. Summarizing

16. The preparation of financial statements which include the statement of financial position, income
statement, statement of comprehensive income, statement of changes in equity and statement of
cash flows is
a. Analyzing c. Classifying
b. Recording d. Summarizing

17. The sorting or grouping of similar and interrelated economic transactions and events into their
respective classes is
a. Analyzing c. Classifying
b. Recording d. Summarizing

18. S1. The documents that report financial information about an entity to decision makers is called
financial statements
S2. Accounting is an information system that measures business activities, processes information into
reports and communicates the reports to decision makers.
a. True, True c. False, True
b. True, False d. False, False
19. S1. The overall objective of accounting is to provide quantitative financial information about a
business that is useful to statement users particularly owners and creditor in making economic
decisions.
S2. An accountant’s primary task is to supply financial information so that the statement users could
make informed judgment and better decision
a. True, True c. False, True
b. True, False d. False, False

20. Republic Act 9298 is known as the


a. Revised Accountancy Law
b. Code of Ethics for Professional Accountants
c. Philippine Accountancy Law of 2004
d. Philippine Accountancy Act of 2004

21. Which of the following is not an objective of Republic Act 9298?


a. The standardization and regulation of accounting education
b. The examination for registration of certified public accountants
c. The supervision, control, and regulation of the practice of accountancy in the Philippines
d. The development and improvement of accounting standards that will be generally accepted in
the Philippines

22. The practice of Accountancy shall include but not limited to the following:
I. Practice of Public Accountancy
II. Practice in Commerce and Industry
III. Practice in Education/Academe
IV. Practice in the Government
a. I only c. I, II and III only
b. I and II only d. I, II, III and IV

23. A CPA is in public accounting practice when he/she


a. Represents his/her employer before government agencies on tax and other matters related to
accounting.
b. Represents his/her clients before government agencies on tax and other matters related to
accounting.
c. Teaches accounting, auditing, management advisory services, accounting aspect of finance,
business law, taxation and other technically related subjects.
d. Holds, or is appointed to, a position in an accounting professional group in government or in a
government-owned and/or controlled corporation where decision making requires professional
knowledge in the science of accounting.

24. Section 4 of the Rules and Regulations Implementing RA 9298 (IRR) provides that any position in any
business or company in the private sector which requires supervising the recording of financial
transactions, preparation of financial statements, coordinating with external auditors for the audit of
such financial statements, and other related functions should be occupied by a duly registered CPA.
It provides further that
a. the business or company where such position exists has a paid-up capital of at least P 5,000,000
and/or annual revenue of at least P 10,000,000.
b. the business or company where such position exists has a paid-up capital of at least P 10,000,000
and/or annual revenue of at least P 5,000,000.
c. the section applies to all incumbents to the position.
d. the section applies only to persons to be employed after the effectivity of the IRR of RA 9298.

25. Which of the following statements concerning the practice of accountancy in the academe/education
is incorrect?
a. CPAs are allowed to teach business law subjects.
b. Members of the Integrated Bar of the Philippines are not allowed to teach business law and
taxation subjects.
c. The position of either the Dean or the Department Chairman or its equivalent that supervises the
Bachelor of Science in Accountancy program of an educational institution is deemed to be in
practice of accountancy in the academe/education.
d. The position of either the Dean or the Department Chairman or its equivalent that supervises the
Bachelor of Science in Accountancy program of an educational institution must be occupied only
by a duly registered CPA.
26. Which of the following statements concerning the practice of accountancy in commerce and industry
is incorrect?
a. A CPA in the practice of accountancy in commerce and industry when he/she involved in decision
making professional knowledge in the science of accounting, as well as the accounting aspects of
finance and taxation.
b. A CPA in the practice of accountancy in commerce and industry when he/she represents his/her
employer before government agencies on tax and other matters related to accounting.
c. A CPA in the practice of accountancy in commerce and industry when such employment or
position requires that the holder thereof must be a Certified Public Accountant
d. A CPA in the practice of accountancy in commerce and industry when he/she renders
professional services as a Certified Public Accountant to more than one client on a fee basis.

27. Which of the following statements concerning the practice of accountancy is correct?
a. A CPA in public accountancy when he/she renders his/her professional services as a CPA to
more than one client on a fee basis in the design, installation, review, and revision of accounting
systems and controls.
b. A CPA in the practice of accountancy in the academe/education when he/she is in an educational
institution teaching any of the subjects included in the Bachelor of Science in Accountancy
curriculum.
c. A CPA is in the practice of accountancy in commerce and industry when his/her position requires
civil service eligibility as a Certified Public Accountant.
d. A CPA is in the practice of accountancy in the government when he/she is involved in the audit or
verification of financial transactions and accounting records of his/her clients.

28. What is the body authorized by law to promulgate rules and regulations affecting the practice of the
accountancy profession in the Philippines?
a. Professional Regulatory Board of Accountancy
b. Philippine Institute of Certified Public Accountants [Accredited Professional Organization (APO)]
c. Securities and Exchange Commission
d. Financial Reporting Standards Council

29. S1. No person shall practice accountancy in this country, or use the title "Certified Public Accountant",
or use the abbreviated title "CPA" or display or use any title, sign, card, advertisement or other device
to indicate such person practices or offers to practice accountancy, or is a certified public accountant,
unless such person shall have received from the BOA a certificate of registration/Professional license
and be issued a professional identification card or a valid temporary/special permit duly issued to
him/her by the BOA and the PRC.
S2. Single practitioners and partners of partnerships for the practice of public accountancy shall be
registered certified public accountants in the Philippines: Provided, That from the effectivity of RA
9298, a certificate of accreditation shall be issued to certified public accountant in public practice only
upon showing, in accordance with rules and regulations promulgated by the BOA and approved by
the PRC, that such registrant has acquired a minimum of three (3) years meaningful experience in any
of the areas of public practice including taxation: Provide, further, that this requirement shall not
apply to those already granted a certificate of accreditation prior to the effectivity of RA 9298. The
Securities and Exchange Commission shall not register any corporation organized for the practice of
public accountancy.
a. True, True c. False, True
b. True, False d. False, False
30. A “meaningful experience” shall be considered as satisfactory compliance with the requirements of
Section 28 of RA 9298 if it is earned in any following provided that such meaningful experience shall
certified under oath by the employer where such meaningful experience was obtained:
I. Commerce and industry and shall include significant involvement in general accounting,
budgeting, tax administration, internal auditing, liaison with external auditors, representing
his/her employer before government agencies on tax and matters to accounting or any other
related functions
II. Academe/education and shall include teaching for at least three (3) trimester or two (2)
semesters subjects in either financial accounting, business law and tax, auditing problems,
auditing theory, financial management and management services. Provided, that the
accumulated teaching experience on these subjects shall not be less than three (3) school
years
III. Government and shall include significant involvement in general accounting, budgeting, tax
administration, internal auditing, liaison with the Commission on Audit or any related
functions
IV. Public Practice and shall include at least one year as audit assistant and at least two years as
auditor in charge of audit engagement covering full audit functions of significant clients
a. I only c. I, II or III only
b. I or II only d. I, II, III or IV

31. This field of public accountancy engages with the examination of financial statements by independent
CPA for the purpose of expressing an opinion as to the fairness with which the financial statements
are prepared.
a. Taxation c. Management Advisory Services
b. External Auditing d. Financial Accounting

32. This field of public accountancy includes the preparation of annual income tax returns and
determination of tax consequences of certain proposed business endeavors. CPAs in this field does
not frequently represents the client in tax investigations. CPAs in this field must have thorough
knowledge with the tax laws and regulation and updated with changes in taxation law and court cases
concerned with interpreting taxation law.
a. Taxation c. Management Advisory Services
b. External Auditing d. Financial Accounting

33. This field of public accountancy generally refers to services to client on matters of accounting, finance,
business policies, organization procedures, product costs, distribution and many other phases of
business conduct and operations.
a. Taxation c. Management Advisory Services
b. External Auditing d. Financial Accounting

34. S1. Republic Act No. 10963 is the law mandating and strengthening the continuing professional
development program for all regulated professions, including the accountancy profession
S2. Under the new BOA Resolution, all CPAs regardless of area or sector of practice shall require to
comply with 120 credit hours required for the renewal of CPA license and accreditation of a CPA to
practice accountancy profession in a compliance period of three years.
a. True, True c. False, True
b. True, False d. False, False

35. This refers to the mandatory inculcation and acquisition of advanced knowledge, skill, proficiency,
and ethical and moral values after initial registration of the CPA for assimilation into professional
practice and lifelong learning.
a. Continuing Professional Development
b. Continuing Professional Learning
c. Continuing Professional Enhancement
d. Continuing Professional Advancement

36. A CPA shall be permanently exempted from 120 credit hours required for the renewal of CPA license
and accreditation of a CPA to practice accountancy profession in a compliance period of three years
upon reaching the age of
a. 55 c. 60
b. 56 d. 65
37. The initial implementation of the 120 credit hours required for the renewal of CPA license and
accreditation of a CPA to practice accountancy profession in a compliance period of three years is
gradual in the following period:

80 credit units 100 credit units 120 credit units


a. 2015 2016 2017
b. 2016 2017 2018
c. 2017 2018 2019
d. 2018 2019 2020

38. S1. Generally accepted accounting principles represent the rules, procedures, practice and standards
followed in the preparation of financial statements.
S2. The process of establishing GAAP is a political process which incorporated political actions of
various interested user groups as well as professional judgment, logic and research.
a. True, True c. False, True
b. True, False d. False, False

39. Which of the following statements regarding accounting theory is incorrect?


a. Accounting concepts are human-made.
b. Accounting concepts are components of accounting theory.
c. Accounting theory has developed primarily in response to government regulations.
d. Accounting theory can be defined as a coherent set of hypothetical, conceptual and pragmatic
principles that form a general frame of reference for a field of inquiry.

40. Which of the following is not directly involved in the accounting standard-setting “due process” in the
Philippines?
a. Board of Accountancy
b. Bureau of Internal Revenue
c. Professional Regulation Commission
d. Financial Reporting Standards Council

41. S1. The overall purpose of accounting standards is to identify proper accounting practices for the
preparation and presentation of financial statements.
S2. A set of high-quality accounting standards is a necessity to ensure comparability and uniformity in
financial statements based on the same financial information.
a. True, True c. False, True
b. True, False d. False, False

42. Any persons applying for examination shall establish the requisites to the satisfaction of the Board
that he/she:
I. is Filipino citizen
II. is of good moral character
III. is a holder of the degree of Bachelor of Science in Accountancy conferred by a school, college,
academy or institute duly recognized and/or accredited by the CHED or other authorized
government offices
IV. has not been convicted of any criminal offense involving moral turpitude
a. I only c. I, II and III only
b. I and II only d. I, II, III and IV
43. Under Resolution No. 262 Series of 2015 of the Professional Regulatory Board of Accountancy, the
following subjects in the CPA Licensure Examination includes the following
I. Theory of Accounts
II. Financial Accounting and Reporting
III. Practical Accounting Problems I
IV. Advance Financial Accounting and Reporting
V. Practical Accounting Problems II
VI. Management Advisory Services
VII. Management Services
VIII. Auditing
IX. Auditing Theory
X. Taxation
XI. Auditing Problems
XII. Regulatory Framework for Business Transaction
XIII. Business Law and Taxation
a. I, III, V, VII, IX, XI and XIII c. I, II, V, VII, VIII, X and XIII
b. II, IV, VI, VIII, X and XII d. I, III, IV, VI, VII and XI

44. S1. To be qualified as having passed the licensure examination for accountants, a candidate must
obtain a general average of seventy-five percent (75%) with no grades lower than sixty-five percent
(65%).
S2. In the event a candidate obtains the rating of seventy-five percent (75%) and above in at least a
majority of subjects he/she shall receive a conditional credit for the subjects passed.
a. True, True c. False, True
b. True, False d. False, False

45. S1. A candidate who received a conditional credit for the subjects passed shall take an examination in
the remaining subjects within three (3) years from the preceding examination.
S2. A candidate who received a conditional credit for the subjects passed and took another
examination within the prescribed period fails to obtain at least a general average of seventy-five
percent (75%) and a rating of at least sixty-five percent (65%) in each of the subjects reexamined,
he/she shall be considered as failed in the entire examination.
a. True, True c. False, True
b. True, False d. False, False

46. A candidate for CPA Licensure examination obtained the following grades: 100, 74, 73, 72, 71, 65.
He/She:
a. Passed c. Conditioned
b. Failed d. Retired

47. A candidate for CPA Licensure examination obtained the following grades: 100, 100, 100, 100, 74, 64.
He/She:
a. Passed c. Conditioned
b. Failed d. Retired

48. A candidate for CPA Licensure examination obtained the following grades: 85, 74, 75, 74, 74, 65.
He/She:
a. Passed c. Conditioned
b. Failed d. Retired

49. S1. The Board of Accountancy shall submit to the Commission the ratings obtained by each candidate
within ten (10) calendar days after the examination, unless extended for just cause.
S2. Any candidate who fails in two (2) complete Certified Public Accountant Board Examinations shall
be disqualified from taking another set of examinations unless he/she submits evidence that he/she
enrolled in and completed at least twenty-four (24) units of subject given in the licensure examination.
a. True, True c. False, True
b. True, False d. False, False
50. S1. In the Philippines, the development of generally accepted accounting principles is formalized
through the creation of Financial Reporting Standards Council (FRSC)
S2. FRSC is a body created by the Board of Accountancy by the recommendation of Professional
Regulation Commission to assist the commission in carrying out its powers and functions provided
under RA 9298.
a. True, True c. False, True
b. True, False d. False, False

51. S1. The accounting standards promulgated by the FRSC constitute the “high hierarchy” of generally
accepted accounting principles in the Philippines.
S2. Philippine Accounting Standards or PAS and Philippine Financial Reporting Standards or PFRS are
the approved statements of the FRSC.
a. True, True c. False, True
b. True, False d. False, False

52. S1. The FRSC is composed of 15 members and a Chairman who had been or is presently a senior
accounting practitioner.
S2. Any member of the Accounting Standards Council shall be qualified to be appointed to the FRSC
a. True, True c. False, True
b. True, False d. False, False

53. S1. The Philippine Interpretation Committee of PIC was formed by the ASC in August 2006 and has
replaced the Interpretations Committee or IC formed by FRSC in May 2000.
S2. The role of the PIC is to prepare interpretations of PFRS for approval by the FRSC and to provide
timely guidance on financial reporting issues specifically addressed in current PFRS.
a. True, True c. False, True
b. True, False d. False, False

54. The name that is presently used for standards issued by the International Accounting Standards Board
(IASB) is
a. International Accounting Standards (IAS)
b. International Financial Reporting Standards (IFRS)
c. International Financial Accounting Interpretations (IFAI)
d. International Generally Accepted Accounting Principles (IGAAP)

55. Which of the following is responsible for setting International Financial Reporting Standards?
a. Financial Accounting Committee
b. Financial Accounting Standards Board
c. International Accounting Standards Board
d. International Accounting Standards Committee

56. The process of establishing financial accounting standards


a. is a legislative process based on rules promulgated by government agencies.
b. is based solely on economic analysis of the effect standard will have if it is implemented.
c. is a democratic process in that a majority of practicing accountants must agree with standard
before it becomes implemented.
d. is a social process which incorporates political actions of various interested user groups as well
as professional research and logic.

57. The international Accounting Standards (IAS) are


a. focused on quantitative rules.
b. based on regulations not concepts.
c. principles-based rather than rules-based.
d. rules-based rather than principles-based.

58. The purpose of the International Financial Reporting Standards is to


a. promote uniform accounting standards among the countries of the world
b. arbitrate accounting disputes between auditors and international entities
c. issue enforceable standards which regulate the financial reporting of multinational entities
d. develop a uniform currency in which the financial transactions of entities throughout the world
would be measured.
59. The Conceptual Framework is intended to assist
a. CPAs in public practice
b. Users of financial information
c. Financial Reporting Standards Council
d. All of these

60. Which is the basic purpose of the Conceptual Framework of Financial Reporting?
a. To develop a single set of high quality IFRS.
b. To promulgate rules and regulations affecting the practice of Philippine Accountancy Profession.
c. To address accounting issues with divergent and unacceptable treatments in the absence of an
authoritative guidance issued by FRSC
d. To assist preparers of financial statements in applying accounting standards and in dealing with
issues that have yet to form the subject of accounting standards

61. What is the authoritative status of the Conceptual Framework?


a. The framework applies when FRSC develops new or revised Standards. An enterprise is never
required to consider this framework.
b. It has the highest level of authority. In case of conflict between the Framework and the Standard
or Interpretation, The Framework overrides the Standard or Interpretation.
c. If there is a Standard or Interpretation that specifically applies to a transaction, it overrides the
Framework. In the absence of a Standard or an Interpretation that specifically applies, the
Framework should be followed.
d. If there is a Standard or Interpretation that specifically applies to a transaction, management
should consider the applicability of Framework in developing and applying an accounting policy
which results in information that is relevant and reliable.

62. Which of the following is the first step within the hierarchy of guidance to which management refers,
and whose applicability at considers, when selecting accounting policies?
a. Apply the requirements in PFRS dealing with similar and related issues
b. Apply a standard from PFRS if it specifically relates to the transaction, event, or condition.
c. Consider the applicability of definitions, recognition criteria, and measurement concepts in the
Conceptual Framework.
d. Consider the most recent pronouncements of other standard-setting bodies to the extent they do
not conflict with PFRS or the Conceptual Framework

63. What statements are intended to meet the needs of users who are not in a position to require an
entity to prepare reports tailored to their particular information needs?
a. Separate financial statements c. Business entity financial statements
b. Consolidated financial statements d. General purpose financial statements

64. Which of the following best states the purpose of general-purpose financial statements?
a. To identify shareholders
b. To help users make decisions.
c. To determine compliance with tax laws.
d. To disclose the market value of the firm.

65. Under Revised Conceptual Framework for Financial Reporting, the objective of general purpose
financial reporting is to provide financial reporting about the reporting entity that is useful to
a. existing and potential investors.
b. existing investors, lenders and other creditors.
c. potential investors, lenders and other creditors.
d. existing and potential investors, lenders and other creditors.

66. A primary objective of financial reporting is to assist


a. investors in analyzing the company
b. investors in predicting prospective cash flows
c. banks to determine an appropriate interest rate for their commercial loans
d. suppliers in determining an appropriate discount to offer a particular company
67. The following are the Scope of the Revised Conceptual Framework
I. Objective of financial reporting
II. Qualitative characteristic of useful financial information
III. Financial statements and reporting entity
IV. Elements of financial statements
V. Recognition and derecognition
VI. Measurement
VII. Presentation and disclosure
VIII. Concepts of capital and capital maintenance
IX. Limitations of financial reporting
a. I, II, III, IV, V, VI, VII, VIII and IX c. I, II, III, IV, V, VI and VII
b. I, II, III, IV, V, VI, VII and VIII d. I, II, III, IV, V and VI

68. The following are objectives of financial reporting, which is not?


I. To provide information that is useful to management
II. To provide information about those investing in the entity
III. To provide information that is useful in making investing and credit decisions
a. I only c. I, II and III
b. I and II only d. none of the choices

69. The term “assessing cash flow prospects” as an objective of financial reporting means
a. cash basis is preferred over accrual basis of accounting
b. over the long run, trends in revenue and expenses are generally more meaningful than trends
in cash receipts and payments
c. information about the financial effects of cash receipts and cash payments is considered the best
indicator of continuing ability to generate favorable cash flows
d. All of these are correct regarding assessing cash flow prospects

70. S1. Liquidity is the availability of cash over a long term to meet financial commitments when they fall
due.
S2. Solvency is the availability of cash in the near future to cover currently maturing obligations.
a. True, True c. False, True
b. True, False d. False, False

71. What are the qualitative characteristics of financial statements according to the Framework?
a. Qualitative characteristics are broad classes of financial effects of transactions and other events.
b. Qualitative characteristics are the attributes that make the information provided in financial
statements useful to others.
c. Qualitative characteristics measure the extent to which an entity has complied with all relevant
Standards and Interpretations.
d. Qualitative characteristics are non-quantitative aspects of an entity’s position and performance
and changes in financial position.

72. Which term best describes information that influences the economic decision of users?
a. Prospective c. Reliable
b. Relevant d. Understandable

73. It is an entity-specific aspect of relevance based on the nature and magnitude, or both, of the items
to which the information relates in the context of an individual entity’s financial report.
a. Comparability c. Feedback value
b. Confirmatory value d. Materiality

74. If financial information that is presented in a balance sheet or income statement is misstated and it
influences the economic decisions of user, that information is described as
a. Faithful c. Prudent
b. Material d. Reliable

75. Under PFRS Conceptual Framework, which of the following is considered a fundamental characteristic
rather than an enhancing characteristic of financial information?
a. Faithful representation c. Understandability
b. Timeliness d. Verifiability
76. Under the Revised Conceptual Framework, which of the following fundamental characteristics
replaces the “reliability” characteristic under the old Conceptual Framework?
a. Faithful representation c. Substance over form
b. Prudence d. Verifiability

77. Which of the following is not an ingredient of “faithful representation” characteristic of financial
information based on the Conceptual Framework of Financial Reporting?
I. Completeness IV. Prudence
II. Confirmatory Value V. Neutrality
III. Free from Error
a. I and III c. I, III, V
b. II and IV d. IV only

78. Under the Revised Conceptual Framework, the following statements are features of financial
information’s comparability characteristics except one
a. Comparability is uniformity
b. A comparison requires at least two items
c. Consistency, although related to comparability, is not the same
d. Comparability is the goal, consistency helps to achieve the goal

79. Which of the following is correct regarding the qualitative characteristic of “understandability” in
relation to information in financial statements?
a. Financial statements should exclude complex matters
b. Financial statements should be free from material error
c. Users are expected to have significant business knowledge
d. Users should be willing to study the information with reasonable diligence

80. What is basic underlying assumption?


a. The financial statements are complete, neutral and free from error
b. The financial statements have predictable value and confirmatory value
c. The financial statements are comparable, understandable, verifiable and timely
d. The financial statements are normally prepared on the basis that the entity will continue in
operation for the foreseeable future

81. Information is material if its omission or misstatement could influence the economic decision that the
users make on the basis of the financial information about an entity. The factors of materiality are the
following, except:
I. Size
II. Timing
III. Nature
a. I only c. III only
b. II only d. I and II only

82. S1. Completeness is the result of the adequate disclosure standard or the principle of full disclosure.
S2. The standard of adequate disclosure is best described by disclosure of any financial facts significant
enough to influence the judgment of informed users.
a. True, True c. False, True
b. True, False d. False, False

83. S1. An accountant shall disclose a material fact known to him which is not disclosed in the financial
statements but disclosure of which is necessary on order that the financial statements would be
misleading.
S2. Notes to financial statements provide narrative description or disaggregation of the items
presented in the financial statements and information about items that qualifies for the recognition.
a. True, True c. False, True
b. True, False d. False, False

84. S1. A neutral depiction is without bias in the preparation or presentation of financial information.
S2. The Revised Conceptual Framework reintroduced the concept of Prudence. Prudence is the
exercise of care and caution when dealing with the uncertainties in the measurement process such
that assets or income are not understated and liabilities or expenses are not overstated.
a. True, True c. False, True
b. True, False d. False, False
85. S1. Under conservatism, as alternatives exist, the alternative which has the least effect on equity
should be chosen.
S2. In the simplest words, conservatism means “in case of doubt, record any loss and do not record
any gain.”
a. True, True c. False, True
b. True, False d. False, False

86. S1. Free from error means an estimate of an unobservable price or value cannot be determine to be
accurate or inaccurate, however, a presentation of that estimate can be faithful if the amount is
described clearly and accurately as an estimate.
S2. The nature and limitation of the process estimating an observable price or value are explained,
and no errors have been made in selecting and applying an appropriate process for developing the
estimate, it is said that the representation of the estimate is faithful.
a. True, True c. False, True
b. True, False d. False, False

87. S1. Measurement uncertainty arises when monetary amounts in financial reports can be observed
directly and must instead be estimated.
S2. As long as the estimate is clearly and accurately described and explained, a high level of
measurement will affect the usefulness of the financial information.
a. True, True c. False, True
b. True, False d. False, False

88. S1. Representing a legal form that differs from the economic substance of the underlying economic
phenomenon or transaction could result in a faithful representation
S2. Measurement uncertainty can affect faithful representation if the level of uncertainty in providing
an estimate is low.
a. True, True c. False, True
b. True, False d. False, False

89. Which of the following is a fundamental quality of useful accounting information?


a. Comparability c. Materiality
b. Consistency d. Relevance

90. The following are enhancing qualitative characteristics relate to the presentation or form of the
financial information:
I. Verifiability
II. Materiality
III. Understandability
IV. Completeness
V. Consistency
VI. Periodicity
VII. Timeliness
a. I, III, V, and VII only c. I, II, VI and V only
b. I, III, IV, and VII only d. I, II, IV and VI only

91. To achieve faithful representation, the financial statements


a. must have predictive and confirmatory value.
b. are comparable, understandable, verifiable and timely.
c. must be complete, neutral and reasonable free from error.
d. All of these would achieve faithful representation.

92. For the information to be useful, the link between the users and decisions made is
a. materiality. c. reliability.
b. relevance. d. understandability.

93. Where is materiality not used in providing financial information?


a. Determining the level of disclosure.
b. Applying the going concern assumption.
c. Applying the revenue recognition principle.
d. Determining what items to include in the financial statements.
94. What is the concept that supports the issuance of interim reports?
a. Consistency c. Relevance
b. Faithful representation d. Materiality
95. S1. Enhancing qualitative characteristics should be maximized to extent necessary.
S2. Enhancing qualitative characteristics, either individually or collectively, cannot render information
useful if that information is irrelevant or not presented faithfully.
a. True, True c. False, True
b. True, False d. False, False

96. Which of the following is true regarding the cost constraint?


a. Benefit are more difficult to quantify than costs.
b. The IASB seeks inputs on costs and benefits as part of due process.
c. Benefits to preparers may include access to capital at a lower cost.
d. All of the choices are correct.

97. S1. Cost is a pervasive constraint on the information that can be provided by financial reporting.
S2. Under cost constraint, the benefit derived from the information should equal the cost incurred in
obtaining the information.
a. True, True c. False, True
b. True, False d. False, False

98. S1. Financial statements provide information about economic resources of the reporting entity, claims
against the entity and changes in the economic resources and claims.
S2. Financial statements provide financial information to users in assessing future cash flows to the
reporting entity and assessing management stewardship of the entity’s economic resources.
a. True, True c. False, True
b. True, False d. False, False

99. The following are information providers:


I. Statement of financial position – recognition of assets, liabilities and equity.
II. Statement of financial performance – recognition of income and expenses.
III. Other statement and notes – presentation and disclosures of information about recognized
assets, liabilities, equity, income and expenses
IV. Other statement and notes – disclosures of information about unrecognized assets and
liabilities
V. Statement of Cash Flows
VI. Other statement and notes – contribution from equity holders and distribution to equity
holders
VII. Other statement and notes – method, assumption and judgment in estimating amount
presented
a. I, II, III, IV, V, VI and VII c. I, II, III, IV, VI and VII only
b. I, II, and III only d. I, II, III, IV and V only

100. The Revised Conceptual Framework recognizes and following types of financial statement:
I. Consolidated financial statements
II. Combined financial statements
III. Unconsolidated financial statements
IV. Uncombined financial statements
a. I only c. I, II and III only
b. I and II only d. I, II, III and IV

101. Financial statements that are prepared when the reporting entity comprises two or more entities that
are not linked by a parent and subsidiary relationship.
a. Consolidated financial statements c. Unconsolidated financial statements
b. Combined financial statements d. Uncombined financial statements

102. Financial statements that are designed to provide information about the parent’s assets, liabilities,
income and expenses and not about those of the subsidiaries.
a. Consolidated financial statements c. Unconsolidated financial statements
b. Combined financial statements d. Uncombined financial statements

103. Financial statements that provide information about the assets, liabilities, equity, income and
expenses of both the parent and its subsidiaries as a single reporting entity.
a. Consolidated financial statements c. Unconsolidated financial statements
b. Combined financial statements d. Uncombined financial statements
104. S1. A reporting entity is an entity that is not required or chooses to prepare financial statements.
S2. A reporting entity is necessarily a legal entity.
a. True, True c. False, True
b. True, False d. False, False

105. The reporting entity can be a single entity or a portion of an entity, or can comprise more than one
entity. Accordingly, the following can be considered a reporting entity:
I. Individual corporation, partnership or proprietorship
II. The parent alone
III. The parent and its subsidiaries as single reporting entity
IV. Two or more entities without parent and subsidiary relationship as a single reporting entity
V. A reportable business segment of an entity
a. I and II only c. I, II, III and IV only
b. I, II and III only d. I, II, III, IV and V

106. S1. Financial statements cannot include information about transactions and other events that
occurred after the end of reporting period if the information is necessary to meet the general
objective of financial statements.
S2. Interim financial statements are prepared on an interim basis, e.g. three months, six months or
nine months, are not optional but required.
a. True, True c. False, True
b. True, False d. False, False

107. Financial statements are prepared for a specified period of time and generally provide information
about:
I. Assets during the reporting period
II. Liabilities during the reporting period
III. Equity during the reporting period
IV. Income at the end of the reporting period
V. Expenses at the end of the reporting period
a. I, II, III, IV, and V c. IV and V only
b. I, II, and III only d. none of the choices

108. The basic notions or fundamental premises on which the accounting process is based.
a. Conceptual Framework c. International Financial Reporting Standards
b. Accounting Assumptions d. International Accounting Standards

109. S1. The Conceptual Framework for Financial Reporting mentions three assumptions – accounting
entity, time period and monetary unit.
S2. Implicit in accounting, the only one assumption – going concern.
a. True, True c. False, True
b. True, False d. False, False

110. Which of the following statements is/are false regarding the Conceptual Framework of Financial
Reporting?
I. The Conceptual Framework does not in any way assist preparers of financial statements in
applying PFRS and in dealing with topics that have yet to form the subject of PFRS.
II. The Conceptual Framework is not a PFRS, and nothing in it overrides any specific PFRS,
including PFRS that is in some respect in conflict with the Conceptual Framework.
III. The Conceptual Framework serves as a guide in developing future financial reporting
standards and in reviewing existing ones.
IV. The Conceptual Framework is a source of guidance for determining an accounting treatment
where a standard does not provide specific guidance.
a. I only c. I, II and III only
b. I and II only d. All of these are correct statements

111. Identify the pervasive constraint and underlying assumption mentioned in the Conceptual
Framework.
A. B. C. D.
Pervasive Constraint Cost Cost Timeliness Timeliness
Underlying Assumption Accrual basis Going concern Accrual basis Going concern
112. S1. An economic resource is a right that has the potential to produce economic benefits.
S2. An obligation is a duty or responsibility that the entity has no practical ability to avoid.
a. True, True c. False, True
b. True, False d. False, False

113. S1. Under accounting entity assumption, the entity is separate from the owners, managers, and
employees who constitute the entity.
S2. When parent and subsidiary relationship exists, consolidated statements for the affiliates are
usually made because for practical and economic purposes the parent and the subsidiary are a “single
economic entity”.
a. True, True c. False, True
b. True, False d. False, False

114. S1. The time period assumption requires that the indefinite life of an entity is subdivided into
accounting periods which are usually of equal length for the purpose of preparing financial reports on
financial position, performance, and cash flows.
S2. A natural business year is a twelve-month period that ends on any month when the business is at
the highest and experiencing slack season.
a. True, True c. False, True
b. True, False d. False, False

115. S1. The monetary unit assumption has two aspect, namely quantifiability and stability of the peso.
Quantifiability aspect means that the purchasing power is constant and that consistency is
insignificant therefore may be ignored.
S2. Stability of the peso assumption means that assets, liabilities, equity, income and expenses should
be stated in terms of unit of measure.
a. True, True c. False, True
b. True, False d. False, False

116. S1. The elements of financial statements refer to the quantitative and qualitative information
reported in the statement of financial position and income statement.
S2. The Conceptual Framework identifies elements that are unique to the statement of changes in
equity.
a. True, True c. False, True
b. True, False d. False, False

117. The following are elements of financial statements


I. Assets
II. Liability
III. Equity
IV. Income
V. Expense
a. I, II and III only c. I, II, III, IV and V
b. IV and V only d. I, II, III, IV and V are not elements of financial statements

118. S1. Under the Old Conceptual Framework, an asset is defined as a present economic resource
controlled by the entity as a result of past events. An economic resource is a right that has the
potential to produce economic benefits.
S2. Under the Revised Conceptual Framework, an asset is defined as a resource controlled by the
entity as a result of past events and from which future economic benefits are expected to flow to the
entity.
a. True, True c. False, True
b. True, False d. False, False

119. Under the Revised Conceptual Framework, the following are essential characteristic of asset:
I. The asset is a present economic resource.
II. The economic resource is a right that has the potential to produce economic benefits.
III. The economic resource is controlled by the entity as a result of past events
IV. The potential economic benefits are expected to flow to the entity.
a. I only c. I, II and III only
b. I and II only d. I, II, III and IV
120. Rights that have the potential to produce economic benefits may take the following forms:
I. Right that correspond to an obligation of another entity
II. Rights that do not correspond to an obligation of another entity
III. Rights established by contract or legislation
IV. Rights to allow others from controlling economic benefits from the asset.
a. I only c. I, II and III only
b. I and II only d. I, II, III and IV

121. An economic resource could produce economic benefits if an entity is entitled to the following:
I. To receive contractual cash flows
II. To exchange economic resources with another party on favorable terms
III. To produce cash inflows or avoid cash outflows
IV. To receive cash by selling the economic resource
V. To extinguish a liability by transferring an economic resource
a. I, II, III, IV and V c. I, II and III only
b. I, II, III and IV only d. I and II only

122. Which of the following is true about control of an economic resource?


a. Control includes the ability to prevent others from using asset and preventing others from
obtaining the economic benefits from the asset.
b. If there are no legal rights, control is absolutely cannot exist if an entity has other means of
ensuring that no other party can benefit from an asset.
c. An entity controls an asset if it has past ability to indirectly use of asset and obtain economic
outflow from it.
d. All of the statements are false.

123. S1. Under the Revised Conceptual Framework, a liability is defined as a present obligation of the entity
arising from past events, the settlement of which is expected to result in an outflow from the entity
of resources embodying economic benefits.
S2. Under the Old Conceptual Framework, a liability is defined as a present obligation of the entity to
transfer an economic resource as a result of past events. An obligation is a duty or responsibility that
the entity has no practical ability to avoid.
a. True, True c. False, True
b. True, False d. False, False

124. Under the Revised Conceptual Framework, the following are essential characteristics of liability:
I. The entity has an obligation.
II. The obligation is to transfer an economic resource.
III. The obligation is a present obligation that exists as a result of past event
a. I only c. I and II only
b. II only d. I, II and III

125. S1. An obligation is a duty or responsibility that an entity has no practical ability to avoid.
S2. Obligations can either be legal or constructive.
a. True, True c. False, True
b. True, False d. False, False

126. S1. Legal obligations arise from normal business practice, custom and a desire to maintain good
business relations or act in an equitable manner.
S2. Obligations may be constructively enforceable as a consequence of a binding contract or statutory
requirement.
a. True, True c. False, True
b. True, False d. False, False

127. Obligations to transfer economic resource includes the following:


I. Obligation to pay cash
II. Obligation to deliver goods or noncash resources
III. Obligation to provide services at some future time
IV. Obligation to exchange economic resources with another party on unfavorable terms
V. Obligation to transfer an economic resource if specified uncertain future event occurs
a. I, II, III, IV and V c. I, II and III only
b. I, II, III and IV only d. I and II only
128. The Revised Conceptual Framework discusses how the ‘no practical ability to avoid’ criterion is applied
in the following:
I. If a duty or responsibility arises from the entity’s customary practices, published policies or
specific statements—the entity has an obligation if it has no practical ability to act in a manner
inconsistent with those practices, policies or statements.
II. If a duty or responsibility is conditional on a particular future action that the entity itself may
take—the entity has an obligation if it has no practical ability to avoid taking that action.
a. I only c. I and II
b. II only d. none of the choices

129. S1. Under the Revised Conceptual Framework, expense is defined as increases in assets, or decreases
in liabilities, that result in increases in equity, other than those relating to contributions from holders
of equity claims.
S2. Under the Revised Conceptual Framework, income is defined as decreases in assets, or increases
in liabilities, that result in decreases in equity, other than those relating to distributions to holders of
equity claims.
a. True, True c. False, True
b. True, False d. False, False

130. Under the Revised Conceptual Framework, this statement refers to the statement of profit or loss and
a statement presenting other comprehensive income.
a. Statement of Financial Position
b. Income Statement
c. Statement of Comprehensive Income
d. Statement of Financial Performance

131. Under the Revised Conceptual Framework, this is the process of capturing for inclusion in the
statement of financial position or the statement(s) of financial performance an item that meets the
definition of an asset, a liability, equity, income or expenses.
a. Recording c. Pre-recognition
b. Recognition d. Preinclusion

132. S1. Recognition of an item results in relevant information may be affected by low probability of a low
economic benefits and existence uncertainty.
S2. Recognition of an item results in a faithful representation may be affected by measurement
uncertainty, recognition inconsistency, and presentation and disclosure.
a. True, True c. False, True
b. True, False d. False, False

133. S1. The expense recognition principle is the application of the matching principle.
S2. The matching principle requires that those costs and expenses incurred in earning a revenue shall
be reported in the same period.
a. True, True c. False, True
b. True, False d. False, False

134. The following are applications of matching principle, except:


a. Cause and effect association
b. Systematic and rational allocation
c. Immediate recognition
d. Derecognition

135. Under the application of matching principle, this involves the simultaneous or combined recognition
of revenue and expenses that result directly and jointly from the same transactions or events.
a. Cause and effect association
b. Systematic and rational allocation
c. Immediate recognition
d. Derecognition
136. Under the application of matching principle, the cost incurred is expense outright because of
uncertainty of future economic benefits or difficulty of reliably associating certain costs with future
revenue.
a. Cause and effect association
b. Systematic and rational allocation
c. Immediate recognition
d. Derecognition

137. Under the application of matching principle, the reason is that the cost incurred will benefit future
periods and there is an absence of direct or clear association of the expense with specific revenue.
a. Cause and effect association
b. Systematic and rational allocation
c. Immediate recognition
d. Derecognition

138. The following are historical costs bases:


I. historical cost provides information derived, at least in part, from the price of the transaction
or other event that gave rise to the item being measured
II. historical cost of assets is reduced if they become impaired and historical cost of liabilities is
increased if they become onerous
III. one way to apply a historical cost measurement basis to financial assets and financial liabilities
is to measure them at amortized cost
a. I only c. III only
b. II only d. I, II and III

139. The following are current value measurement bases:


I. current value provides information updated to reflect conditions at the measurement date
II. current value measurement bases include fair value, value in use (assets) fulfillment value (for
liabilities and current cost.
a. I only c. I and II
b. II only d. none of the choices

140. Adding together of assets, liabilities, equity, income and expenses that have similar or shared
characteristics and are included in the same classification is called
a. Classification c. Duplication
b. Segregation d. Aggregation

141. S1. Return of capital is the amount in excess of their original investment
S2. Return on Capital is an erosion of the capital invested in the entity
S3. Capital maintenance approach means that net income occurs only after the capital used from the
beginning of the period is maintained
a. True, True, True c. False, True, False
b. True, False, False d. False, False, True

142. This is the monetary amount of the net assets contributed by shareholders and the amount of the
increase in net assets resulting from earnings retained by the entity.
a. Shareholders’ Equity c. Legal Capital
b. Financial Capital d. Physical Capital

143. Which of the following statements is incorrect in relation to fair presentation?


a. An entity shall not describe financial statements as complying with PFRS unless they comply with
all the requirements of PFRS.
b. An entity whose financial statements comply with PFRS shall make an explicit and unreserved
statement of such compliance in the notes.
c. An entity can rectify inappropriate accounting policies either by disclosure of the accounting
policies used by notes or explanatory material.
d. Fair presentation requires the faithful representation of the effects of transactions in accordance
with the definition criteria for assets, liabilities, income and expenses.

144. Which of the following is not an acceptable major asset classification?


a. Current assets c. Investments
b. Deferred charges d. Property, plant and equipment
145. The basis for classifying assets as current or noncurrent is conversion to cash within the
a. operating cycle or one year, whichever is longer.
b. operating cycle or one year, whichever is shorter.
c. accounting cycle or one year, whichever is longer.
d. accounting cycle or one year, whichever is shorter.

146. “To provide information about the financial position, financial performance and cash flows of an entity
that is useful to a wide range of users in making economic decisions,” is the objective of
a. Statement of financial performance c. Financial statements
b. Statement of cash flows d. Financial Reporting

147. S1. Financial statements shall be presented at most annually.


S2. Financial statements show the result of the management stewardship of the resources entrusted
to it.
a. True, True c. False, True
b. True, False d. False, False

148. When an entity’s end of reporting period changes and financial statements are presented for a period
longer or shorter than one year, an entity shall disclose:
I. The period covered by the financial statements.
II. The reason for using a longer or shorter period.
III. The fact that amounts presented in the financial statements are not entirely comparable.
IV. The value of amounts affected by the change.
a. I only c. I, II and III only
b. I and II only d. I, II, III and IV

149. S1. When an entity is functioning with a clearly identifiable operating cycle, the separate classification
of current and noncurrent assets is useful information by distinguishing between net assets that are
continuously circulating as working capital from the net assets, used in long-term operations.
S2. When the entity’s normal operating cycle is clearly identifiable, the duration is presumed to be
twelve months.
a. True, True c. False, True
b. True, False d. False, False

150. The asset restricted to settle a liability for more than twelve months after the reporting period is
a. Cash c. Current Asset
b. Cash Equivalent d. Other Noncurrent Asset

151. S1. A statement of financial performance is a formal statement showing the following three elements:
assets, liabilities and equity.
S2. Investor, creditors and other statement users analyze the statement of financial performance to
evaluate such factors as liquidity, solvency and the need of the entity for additional financing.
a. True, True c. False, True
b. True, False d. False, False

152. S1. PAS 1 does not apply to the structure and content of condensed interim financial statements
prepared in accordance with PAS 34 Interim Financial Reporting.
S2. If entities with not-for-profit activities in the private sector or the public sector apply PAS 1, they
may need to amend the descriptions used for particular line items in the financial statements and for
the financial statements themselves.
a. True, True c. False, True
b. True, False d. False, False

153. S1. Under PAS 1 paragraph 66, simply states that “an entity shall classify all other assets not classified
as current as either current or noncurrent depending on its life expectancy”.
S2. Property, plant and equipment are tangible assets which are held by an entity for use in
production, or supply of goods and services, for rental to others, administrative, purposes and are
expected to be used during more than on period.
a. True, True c. False, True
b. True, False d. False, False
154. S1. IASC defines long-term investment as “an asset held by an entity for the accretion of wealth
through capital distribution, such as interest, royalties, dividends and rentals, for capital appreciation
or for other benefits to the investing entity such as those obtained through trading relationship”.
S2. Intangible asset is an identifiable nonmonetary asset without substance. Identifiable asset includes
patent, franchise, copyright, lease right, goodwill, trademark and computer software.
a. True, True c. False, True
b. True, False d. False, False

155. An entity must normally present a classified statement of financial position, separating current and
non-current assets and liabilities, unless presentation based on liquidity provides information that is
reliable. Current assets are assets that are:
I. expected to be realized in the entity's normal operating cycle
II. held primarily for the purpose of trading
III. expected to be realized within 12 months after the reporting period
IV. cash and cash equivalents (unless restricted).
a. I only c. I, II and III only
b. I and II only d. I, II, III and IV

156. An entity must normally present a classified statement of financial position, separating current and
non-current assets and liabilities, unless presentation based on liquidity provides information that is
reliable. Current liabilities are those:
I. expected to be settled within the entity's normal operating cycle
II. held for purpose of trading
III. due to be settled within 12 months
IV. for which the entity does not have an unconditional right to defer settlement beyond 12
months (settlement by the issue of equity instruments does not impact classification).
a. I only c. I, II and III only
b. I and II only d. I, II, III and IV

157. PAS 1, paragraph 54, provides that as minimum, the line items under current assets are:
I. Cash and cash equivalents
II. Financial assets at fair value such as trading securities and other investments in quoted equity
instruments
III. Trade and other receivables
IV. Inventories
V. Prepaid expenses
a. I and II only c. I, II, III and IV only
b. I, II and III only d. I, II, III, IV and V

158. PAS 1, paragraph 54, provides that as minimum, the line items under current liabilities are:
I. Trade and other payables
II. Current provisions
III. Short-term borrowing
IV. Current portion of long-term debt
V. Current tax liability
a. I and II only c. I, II, III and IV only
b. I, II and III only d. I, II, III, IV and V

159. S1. The term “trade and other payables” is a line item for accounts payable, notes payable, accrued
interest on notes payable, dividends payable and accrued expenses. Trade accounts and notes
payable shall not be separately presented.
S2. Under PAS 1, paragraph 69, provides that noncurrent liabilities that not classified as current are
classified as noncurrent.
a. True, True c. False, True
b. True, False d. False, False
160. A liability which is due to be settled within twelve months after the reporting period is classified as
current liabilities, even if:
I. The original term was for a period longer than twelve months.
II. An agreement to refinance or to reschedule payment on a long-term basis is completed
before the reporting period and after the financial statements are authorized for issue.
a. I c. I and II
b. II d. none of the choices

161. If the refinancing on a long-term basis is completed on or before the end of reporting period, the
refinancing is an adjusting event and therefore the obligation is classified as
a. current c. either current or noncurrent
b. noncurrent d. current and noncurrent

162. S1. If the entity has the discretion to refinance or roll over an obligation for at least twelve months
after the reporting period under an existing loan facility, the obligation is classified as current liability.
S2. If the refinancing or rolling over is not at the discretion of the entity, the obligation is classified as
noncurrent liability.
a. True, True c. False, True
b. True, False d. False, False

163. S1. As to effect of breach of covenants, PAS 1, paragraph 74 provides that the liability is classified as
noncurrent even if the lender has agreed, after the reporting period and before the statements are
authorized for issue, not to demand payment as a consequence of the breach.
S2. As to effect of breach of covenants, PAS 1, paragraph 75 provides that the liability is classified as
current if the lender has agreed on or before the end of the reporting period to provide a grace period
ending at least twelve months after the end of the reporting period.
a. True, True c. False, True
b. True, False d. False, False

164. S1. Owners are the holders of instruments classified as equity. The term equity may simply be used
for all business entities.
S2. Shareholders’ equity is the residual interest in the assets of the entity after deducting all of its
liabilities
a. True, True c. False, True
b. True, False d. False, False

165. S1. The IASB concluded that “statement of financial position” better reflects the function of the
statement and is consistent with the conceptual framework.
S2. The IASB concluded that “balance sheet” simply reflected the convention that double-entry
bookkeeping requires all debits to equal credits, and did not identify the content or purpose of the
statement.
a. True, True c. False, True
b. True, False d. False, False

166. The following are elements which are always to be displayed in the heading of a statement of financial
position:
I. The entity whose financial position is being presented
II. The address of the entity whose financial position is being presented
III. The title of the statement
IV. The date of the statement
a. I and II only c. I, III and IV only
b. I and III only d. I, II and IV only

167. The notes to the financial statement must:


I. present information about the basis of preparation of the financial statements and the
specific accounting policies used.
II. disclose any information required by IFRSs that is not presented elsewhere in the financial
statements.
III. provide additional information that is not presented elsewhere in the financial statements
but is relevant to an understanding of any of them.
a. I only c. I and II only
b. II only d. I, II and III
168. PAS 1 suggests that noted should normally be presented in the following order:
I. a statement of compliance with PFRSs.
II. a summary of significant accounting policies applied, including the measurement basis (or
bases) used in preparing the financial statements the other accounting policies used that are
relevant to an understanding of the financial statements.
III. supporting information for items presented on the face of the statement of financial position
(balance sheet), statement(s) of profit or loss and other comprehensive income, statement of
changes in equity and statement of cash flows, in the order in which each statement and each
line item is presented.
IV. other disclosures, including contingent liabilities and unrecognized contractual commitments
non-financial disclosures, such as the entity's financial risk management objectives and
policies.
a. I, II, III, IV c. I, IV, II, III
b. I, III, II, IV d. II, III, IV, I

169. In the Philippines, the common practice is to present in the statement of financial position
a. Current assets before noncurrent assets, noncurrent liabilities before current liabilities and equity
after liabilities.
b. Noncurrent assets before current assets, noncurrent liabilities before current liabilities and equity
after liabilities
c. Current assets before noncurrent assets, current liabilities before noncurrent liabilities and
equity after liabilities
d. Current assets before noncurrent assets, noncurrent liabilities before current liabilities and equity
after liabilities

170. Which statement about the statement of financial position is not true?
a. Biological assets should be reported in the statement of financial position
b. Provisions should be recognized in the statement of financial position
c. A revaluation surplus on a noncurrent asset in the current year should be recognize in the
income statement
d. The number of shares authorized for issue should be reported in the statement of financial
position or the statement of changes in equity or in the notes.

171. Under PAS 1, paragraph 79, An entity shall disclose the following, either in the statement of financial
position or the statement of changes in equity, or in the notes:
I. for each class of share capital – the number of shares authorized, issued and fully paid, and
issued but not fully paid
II. for each class of share capital – par value per share, or that the shares have no par value
III. for each class of share capital – a reconciliation of the number of shares outstanding at the
beginning and at the end of the period
IV. for each class of share capital – the rights, preferences and restrictions attaching to that class
including restrictions on the distribution of dividends and the repayment of capital
V. for each class of share capital – shares in the entity held by the entity or by its subsidiaries or
associates
VI. for each class of share capital – shares reserved for issue under options and contracts for the
sale of shares, including terms and amounts
VII. a description of the nature and purpose of each reserve within equity
a. I, II, III, IV, V, VI and VII c. I, II, III, IV and V only
b. I, II, III, IV, V and VI only d. I, II, III and IV only

172. PAS 1 refers to it as changes in equity other than introduction and return of capital to owners.
a. Net income c. Profit
b. Other comprehensive income d. Total comprehensive income

173. The presence of “cost of sales” account in the income statement signifies that an entity classifies
expenses according to
a. amounts c. maturity
b. function d. nature

174. An entity classified expenses by logistics quality control, manufacturing plant engineering, sales and
marketing, research and development, finance and administration. The classification basis is by
a. area of responsibility c. nature of expense
b. function performed d. object of expenditure
175. If the classification of expenses by function method is used for the presentation of an income
statement, additional information on the following items must be disclosed.
a. Revenue c. Gains on revaluation of assets
b. Gains on disposal of assets d. Depreciation and amortization expense

176. Under PAS 1, which of the following item is not included in the computation of profit?
a. Finance cost
b. Post-tax gain or loss on discontinued operations
c. Unrealized gain in change in value of biological assets
d. Unrealized gain in change in value of available-for-sale securities

177. The following are items recognized as other comprehensive income:


I. Changes in revaluation surplus where the revaluation method is used under PAS 16 Property,
Plant and Equipment and PAS 38 Intangible Assets
II. Remeasurements of a net defined benefit liability or asset recognized in accordance with PAS
19 Employee Benefits
III. Exchange differences from translating functional currencies into presentation currency in
accordance with PAS 21 The Effects of Changes in Foreign Exchange Rates
IV. Gains and losses on remeasuring available-for-sale financial assets in accordance with PAS 39
Financial Instruments: Recognition and Measurement
V. The effective portion of gains and losses on hedging instruments in a cash flow hedge under
PAS 39 or PFRS 9 Financial Instruments
VI. Gains and losses on remeasuring an investment in equity instruments where the entity has
elected to present them in other comprehensive income in accordance with PFRS 9
VII. The effects of changes in the credit risk of a financial liability designated as at fair value
through profit and loss under PFRS 9
a. I, II, III, IV, V, VI and VII c. I, II, III, IV and V only
b. I, II, III, IV, V and VI only d. I, II, III and IV only

178. S1. An entity can apply the two ways of presenting comprehensive income simultaneously, namely:
two statements and single statement of comprehensive income.
S2. Two statement is the combined statement showing the components of profit or loss and
components of other comprehensive income in a single statement.
a. True, True c. False, True
b. True, False d. False, False

179. Under PAS 1, which of the following should be classified as extraordinary item in reporting results of
operations?
a. Losses resulting from unusual major flashflood in the Visayas region
b. Gain resulting from the national government expropriation of a corporate property
c. Foreign exchange losses arising from appreciation of Japanese yen relative to the Philippine peso.
d. None, all are ordinary gains and losses.

180. PAS 1 requires the allocation of profit and loss for the period between or among
I. Profit or loss attributable to owner of the parent.
II. Profit or loss attributable to subsidiaries of the parent.
III. Profit or loss attributable to non-controlling interests.
a. I and II only c. II and III only
b. I and III only d. I, II and III only

181. PAS 1 requires the allocation of total comprehensive income for the period between or among
I. Total comprehensive income attributable to owners of the parent.
II. Total comprehensive income attributable to subsidiaries of the parent.
IV. Total comprehensive income attributable to non-controlling interests.
a. I and II only c. II and III only
b. I and III only d. No such required

182. The following are statements relating to presentation of the statement of changes in equity:
S1. The amounts of dividends shown as distribution to owners and the amounts of dividends per
share should be shown in the notes only.
S2. Components of equity include each class of contributed equity, the accumulated balance of each
class of other comprehensive income and retained earnings.
a. True, True c. False, True
b. True, False d. False, False
183. What is the purpose of reporting comprehensive income?
a. To replace net income with a better measure.
b. To report a measure of overall entity performance.
c. To report changes in equity due to transactions with owners.
d. To combine income from continuing operations with income from discontinued operations.

184. Which is not acceptable in reporting other comprehensive income?


a. Note to financial statements
b. Statement of changes in equity
c. Single statement of comprehensive income
d. Separate statement of comprehensive income

185. Which of the following OCI will be classified to profit or loss:


I. Unrealized gain or loss on equity investment measured at fair value through other
comprehensive income.
II. Unrealized gain or loss on debt instrument measured at fair value through other
comprehensive income.
III. Revaluation surplus during the year.
IV. Gain or loss from translating financial statements of a foreign operation.
V. Remeasurements of defined benefit plan including actuarial gain or loss.
VI. Unrealized gain or loss on derivative contracts designated as cash flow hedge.
VII. Change in fair value attributable to credit risk of a financial liability designated at fair value
through profit or loss.
a. I, II, III, IV, V, VI and VII c. II, IV and VI only
b. I, III, V and VII only d. I, II, III, IV and V only

186. PAS 1, paragraph 85, states that an entity shall present additional line items, headings and subtotals
in the statement(s) presenting profit or loss and other comprehensive income when such presentation
is relevant to an understanding of the entity’s financial performance. In accordance to this those
subtotals shall:
I. be comprised of line items made up of amounts recognized and measured in accordance with
PFRS
II. be presented and labelled in a manner that makes the line items that constitute the subtotal
clear and understandable
III. be consistent from period to period
IV. not be displayed with more prominence than the subtotals and totals required in IFRS for the
statement(s) presenting profit or loss and other comprehensive income
a. I only c. I, II and III only
b. I and II only d. I, II, III and IV

187. The following are sources of income but not limited to:
I. Sales of merchandise to customers
II. Rendering of services
III. Use of entity resources
IV. Disposal of resources other than products
a. I only c. I, II and III only
b. I and II only d. I, II, III and IV

188. In which section of the statement of financial performance should employment taxes that are due for
settlement in 15 months’ time be presented?
a. Current liabilities c. Noncurrent liabilities
b. Current assets d. None of the choices

189. Assets to be sold, consumed or realized as part of normal operating cycle are
a. Revenue c. Selling expense
b. Finance cost d. None of the choices

190. Conceptually, net income/loss is a measure of


a. Wealth c. Capital maintenance
b. Change of wealth d. Cash flow
MAE Company provided the following account balances on December 31, 2019 (all amounts in Philippine
peso: (PROBLEM 8-2)

Preference share redemption fund 350,000


Withholding tax payable 30,000
Ordinary share capital 5,000,000
Dividends payable 120,000
Bonds Payable 5,000,000
Allowance for doubtful accounts 20,000
Accumulated depreciation – equipment 200,000
Accumulated depreciation – building 2,000,000
Accrued salaries 100,000
Accounts payable 400,000
SSS payable 10,000
Notes payable 300,000
Share premium – ordinary 200,000
Preference share capital 2,000,000
Premiums on bonds payable 1,000,000
Share premium – preference 500,000
Retained earnings (deficit) (1,800,000)
Unearned rent income 40,000
Trading securities 280,000
Prepaid expenses 70,000
Computer software 3,250,000
Notes receivable 250,000
Land held for speculation 500,000
Land 1,500,000
Inventories 1,300,000
Accrued interest on notes receivable 10,000
Patent 100,000
Equipment 1,000,000
Cash surrender value 60,000
Cash and cash equivalents 500,000
Long-term refundable deposit 50,000
Building 5,000,000
Sinking fund 400,000
Advances to officers – not currently collectible 100,000
Accounts receivable 400,000

191. MAE Company should report “Trade and other receivables” in the face of its statement of financial
position as
a. Php 630,000 c. Php 650,000
b. Php 640,000 d. Php 660,000

192. MAE Company should report “Property, plant and equipment” in the face of its statement of financial
position as
a. Php 7,500,000 c. Php 5,300,000
b. Php 6,000,000 d. Php 3,800,000

193. MAE Company should report “Long-term investments” in the face of its statement of financial position
as
a. Php 1,250,000 c. Php 1,310,000
b. Php 910,000 d. Php 900,000

194. MAE Company should report “Intangible assets” in the face of its statement of financial position as
a. Php 3,250,000 c. Php 3,350,000
b. Php 3,150,000 d. Php 100,000

195. MAE Company should report “Other noncurrent assets” in the face of its statement of financial
position as
a. Php 150,000 c. Php 200,000
b. Php 100,000 d. Php 50,000
196. MAE Company should report “Trade and other payables” in the face of its statement of financial
position as
a. Php 650,000 c. Php 880,000
b. Php 970,000 d. Php 1,000,000

197. MAE should report “Share capital” in the face of its statement of financial position as
a. Php 7,700,000 c. Php 5,500,000
b. Php 7,000,000 d. Php 2,200,000

198. MAE should report “Reserves” in the face of its statement of financial position as
a. Php 1,050,000 c. Php 850,000
b. Php 700,000 d. Php 1,750,000

199. MAE should report “Total current assets” in the face of its statement of financial position as
a. Php 2,780,000 c. Php 2,800,000
b. Php 2,790,000 d. Php 2,810,000

200. MAE should report “Total noncurrent assets” in the face of its statement of financial position as
a. Php 12,310,000 c. Php 10,000,000
b. Php 10,050,000 d. Php 10,110,000

201. MAE should report “Total noncurrent liabilities” in the face of its statement of financial position as
a. Php 5,000,000 c. Php 5,120,000
b. Php 6,000,000 d. Php 6,120,000

202. MAE should report “Total shareholders’ equity” in the face of its statement of financial position as
a. Php 6,000,000 c. Php 5,900,000
b. Php 5,800,000 d. Php 5,700,000

MJ Company provided the following information for the year 2019:


Income tax expense 320,000
Raw Materials – Jan 1 200,000
Goods in process – Jan 1 240,000
Finished goods – Jan 1 360,000
Raw Materials – Dec 31 280,000
Goods in process – Dec 31 170,000
Finished goods – Dec 31 300,000
Sales 7,500,000
Gain from expropriation of asset 100,000
Earthquake loss 300,000
Office expenses 250,000
Accounting and legal fees 150,000
Delivery expenses 200,000
Gain on sale of equipment 100,000
Interest income 10,000
Sales returns and allowances 50,000
Depreciation – machineries 60,000
Depreciation – store equipment 70,000
Depreciation – office equipment 40,000
Office salaries 150,000
Advertising 160,000
Sales salaries 400,000
Factory supplies used 110,000
Repairs and maintenance – machinery 50,000
Rent – factory building 120,000
Light, heat and power 320,000
Superintendence 210,000
Indirect labor 250,000
Direct labor 950,000
Purchases 3,000,000
203. Under “Cost of goods sold” method, “Net sales revenue” of JM Company to be reported in its Income
Statement is
a. Php 7,500,000 c. Php 7,710,000
b. Php 7,450,000 d. Php 7,660,000

204. Under “Cost of goods sold” method, “Cost of goods sold” of JM Company to be reported in its Income
Statement is
a. Php 5,230,000 c. Php 5,060,000
b. Php 5,420,000 d. Php 5,120,000

205. Under “Cost of goods sold” method, “Distribution costs” of JM Company to be reported in its Income
Statement is
a. Php 830,000 c. Php 870,000
b. Php 980,000 d. Php 910,000

206. Under “Nature of expense” method, “Total income” of JM Company to be reported in its Income
Statement is
a. Php 7,450,000 c. Php 7,500,000
b. Php 7,660,000 d. Php 7,710,000

207. “Net income” of JM Company is Php 500,000 in what method of presentation of Income Statement?
a. Nature of expense method c. a and b
b. Cost of goods sold method d. neither a nor b

208. Which of the following would represent the least likely use of an income statement?
a. Use by investor interested in financial position
b. Use by labor unions to examine earnings closely as a basis for salary discussions
c. Use by customers to determine an entity’s ability to provide needed goods and services
d. Use by government to formulate tax policy

209. Investor and creditors use income statement for all of the following, except
a. To evaluate past performance of an entity
b. To provide basis for predicting future performance
c. To help assess the risk and uncertainty of achieving future cash flows
d. To evaluate the future performance of an entity

210. Which of the following changes during a period is not a component of other comprehensive income?
a. Unrealized gain on equity instrument measured at fair value through other comprehensive
income
b. Actuarial gain on defined benefit plan
c. Foreign currency translation adjustment
d. Treasury share

211. S1. Information about the cash flows of an entity is useful in providing users of financial statements
with a basis to assess the ability of the entity to generate cash and cash equivalents and the needs of
the entity to utilize those cash flows.
S2. The objective of IAS 7 is to require the provision of information about the historical changes in
cash and cash equivalents of an entity by means of a statement of cash flows which classifies cash
flows during the period from operating, investing and financing activities.
a. True, True c. False, True
b. True, False d. False, False

212. S1. Statement of cash flows provides information about cash receipts and cash payments of an entity
during a period.
S2. A statement of cash flows, when used in conjunction with the rest of the financial statements,
provides information that enables users to evaluate the changes in net assets of an entity.
a. True, True c. False, True
b. True, False d. False, False
213. S1. Investing activities are activities that result in changes in the size and composition of the
contributed equity and borrowings of the entity.
S2. Financing activities are the acquisition and disposal of long-term assets and other investments not
included in cash equivalents.
a. True, True c. False, True
b. True, False d. False, False
214. The following are cash flows from operating activities, except
a. Cash payments to acquire equity instruments of other entity
b. Cash receipts for securities held for trading
c. Cash receipts from royalties, rental fees, commissions and other revenue
d. Cash payments for salaries and wages of production workers

215. The following are cash flows from investing activities, except
a. Cash proceeds from sale of land not intended for use and reported as investment property
b. Advances to other parties other than advances made by construction company
c. Cash payments for acquiring tools
d. Cash used to acquire treasury shares

216. The following are cash flows from financing activities, except
a. Cash proceeds from issuing ordinary shares
b. Cash receipts from issuing bonds
c. Cash payments for settling mortgages
d. Cash payments for royalties

217. Which of the following is a noncash transaction?


a. Issuing share capital in exchange of previously issued bonds
b. Reduction to lease payable by issuance and paying cheque
c. Usage of petty cash fund for postage stamps
d. Electronic bank transfer for settlement of previously acquired equipment

MAE Company provided the following information during the current year:
Dividend received 500,000
Dividend paid 1,000,000
Cash received from customers 10,000,000
Cash received from the sale of building 800,000
Proceeds from issuing share capital 1,500,000
Interest received 200,000
Proceeds from sale of long-term investments 2,000,000
Cash paid to suppliers and employees 6,000,000
Interest paid on long-term debt 400,000
Income taxes paid 300,000
Cash payment to purchase land 800,000
Cash balance, January 1 1,800,000

218. What is the net cash provided by operating activities?


a. 4,300,000 c. 3,700,000
b. 4,000,000 d. 3,000,000

219. What is the net cash provided by investing activities?


a. 2,500,000 c. 2,000,000
b. 2,200,000 d. 0

220. What is the net cash provided by financing activities?


a. 1,500,000 c. 500,000
b. 1,000,000 d. 0

221. What is the cash balance on December 31?


a. 8,300,000 c. 6,500,000
b. 7,300,000 d. 5,800,000

222. Under PAS 7, the dividend received from share investments can be classified as
a. Operating activity only c. Either operating or financing activity
b. Financing activity only d. Either operating or investing activity

223. Under Pas 7, an entity can report interest paid on bank loan in the statement of cash flows
a. Either in operating or investing activity c. In financing activity
b. Either in operating or financing activity d. In operating activity
224. Which classification of the cash flow arising from the proceeds of flash flood calamity settlement
would be most appropriate?
a. Operating activities c. Financing activities
b. Investing activities d. Does not appear in the statement of cash flows

225. S1. Unrealized gains and losses arising from changes in foreign currency exchange rates are not cash
flows.
S2. The effect of exchange rate changes on cash and cash equivalents held or due in a foreign currency
is reported in the statement of cash flows in order to reconcile cash and cash equivalents at the
beginning and the end of the period.
a. True, True c. False, True
b. True, False d. False, False

226. S1. It is optional for an entity to outline all significant accounting policies applied in preparing financial
statements.
S2. An entity that does not apply the same accounting policies, generally, does not achieve
comparability of financial statement.
a. True, True c. False, True
b. True, False d. False, False

227. An entity is permitted to change an accounting policy only if the change:


I. is required by a standard or interpretation
II. results in the financial statements providing profitable and feasible operation, other events
or conditions on the entity’s financial statements
III. results in the financial statements providing more relevant and reliable information about the
effects of transactions, other events or conditions on the entity’s financial position,
performance or cash flows
a. I only c. I and III only
b. I and II only d. I, II and III

228. When a standard or an Interpretation specifically applies to a transaction, other event or condition,
the accounting policy or policies applied to that item must be determined by applying the Standard
or Interpretation and considering any relevant Implementation Guidance issued by the FRSC for the
Standard or Interpretation.
In the absence of a Standard or an Interpretation that specifically applies to a transaction, other event
or condition, management must use its judgement in developing and applying an accounting policy
that results in information that is relevant and reliable. In making that judgment, the management
must refer to, and consider the applicability of the following sources:
I. the requirements and guidance in FRSC standards and interpretations dealing with similar and
related issues
II. the definitions, recognition criteria and measurement concepts for assets, liabilities, income
and expenses in the Conceptual Framework
III. the most recent pronouncements of other standard-setting bodies that use a similar
conceptual framework to develop accounting standards, other accounting literature and
accepted industry practices
The order of the aforementioned application should be
a. I before II (III may consider) c. I and II simultaneously (III may consider)
b. II before I (III may consider) d. I and II interchangeably (III may consider)

229. The following are changes in accounting policy, except


a. Change from PFRS for SMEs to Full PFRS reporting.
b. Change in the method of inventory valuation from LIFO to FIFO.
c. Change in depreciation pattern of office equipment, from straight line method to double
declining balance method.
d. Initial adoption with revaluation of carrying amount of assets.
230. The following are disclosures relating to changes in accounting policy caused by a new standard or
interpretation:
I. the title of the standard or interpretation causing the change
II. the nature of the change in accounting policy
III. a description of the transitional provisions, including those that might have an effect on future
periods
IV. for the current period and each prior period presented, to the extent practicable, the amount
of the adjustment for each financial statement line item affected, and for basic and diluted
earnings per share (only if the entity is applying IAS 33)
V. the amount of the adjustment relating to periods before those presented, to the extent
practicable
VI. if retrospective application is impracticable, an explanation and description of how the change
in accounting policy was applied.
a. I, II, III, IV, V and VI c. I, II, III and IV only
b. I, II, III, IV and V only d. I, II and III only

231. S1. A change in accounting estimate is an adjustment of the carrying amount of an asset or liability,
or related expense, resulting from reassessing the expected future benefits and obligations associated
with that asset or liability.
S2. A change in accounting estimate is a normal recurring or adjustment of an asset or liability which
is the natural result of the use of an estimate.
a. True, True c. False, True
b. True, False d. False, False

232. The effect of a change in an accounting estimate shall be recognized currently and prospectively by
including it in profit or loss in:
I. the period of the change, if the change affects the period only
II. the period of the change and future periods, if the change affects both.
a. I and II c. II only
b. I only d. neither I nor II

233. S1. Prospective application means that any resulting adjustment from the change in accounting policy
shall be reported as an adjustment to the opening balance of retained earnings.
S2. Retrospective application means that the change is applied to transactions, other events and
conditions from the date of change in estimate.
a. True, True c. False, True
b. True, False d. False, False

234. Provided are the disclosures relating to changes in accounting estimates:


I. the nature and amount of a change in an accounting estimate that has an effect in the current
period or is expected to have an effect in future periods
II. if the amount of the effect in future periods is not disclosed because estimating it is
impracticable, an entity shall disclose that fact
a. I only c. I and II
b. II only d. neither I nor II

235. S1. The general principle in IAS 8 is that an entity must correct all material prior period errors
retrospectively in the first set of financial statements authorized for issue after their discovery by
restating the comparative amounts for the prior period(s) presented in which the error occurred.
S2. The general principle in IAS 8 is that an entity must correct all material prior period errors
retrospectively in the first set of financial statements authorized for issue after their discovery by
restating the opening balances of assets, liabilities and equity for the earliest prior period presented
if the error occurred before the earliest prior period presented.
a. True, True c. False, True
b. True, False d. False, False

236. The objectives of PAS 10 – Events after the Reporting Period is to prescribe
I. when an entity should adjust its financial statements for events after the reporting period
II. the disclosures that an entity should give about the date when the financial statements were
authorized for issue and about events after the reporting period
III. an entity should not prepare its financial statements on a going concern basis if events after
the reporting period indicate that the going concern assumption is not appropriate
a. I only c. I and III only
b. I and II only d. I, II and III
237. S1. Nonadjusting events after the reporting period are those that provide evidence of conditions that
exist at the end of reporting period.
S2. Adjusting events after the reporting period are those that are indicative of conditions that arise
after the end of reporting period.
a. True, True c. False, True
b. True, False d. False, False

238. An event, which could be favorable or unfavorable, that occurs between the end of the reporting
period and the date that the financial statements are authorized for issue.
a. Events during the reporting period c. Subsequent events
b. Events before the reporting period d. Fortuitous events

239. Non-adjusting events should be disclosed if they are of such importance that non-disclosure would
affect the ability of users to make proper evaluations and decisions. The required disclosure is/are
a. the nature of the event
b. an estimate of its financial effect or a statement that a reasonable estimate of the effect cannot
be made
c. a and b are required disclosures
d. a and b are not required disclosures

240. Adjusting events exclude all, except


a. the receipt of information after the reporting period indicating that an asset was impaired at
the end of the reporting period, or that the amount of a previously recognized impairment loss
for that asset needs to be adjusted
b. a major business combination after the reporting period or disposing of a major subsidiary
c. major ordinary share transactions and potential ordinary share transactions after the reporting
period
d. entering into significant commitments or contingent liabilities

241. Nonadjusting events exclude all, except


a. commencing major litigation arising solely out of events that occurred after the reporting period
b. the determination after the reporting period of the amount of profit-sharing or bonus payments,
if the entity had a present legal or constructive obligation at the end of the reporting period to
make such payments as a result of events before that date
c. the sale of inventories after the reporting period may give evidence about their net realizable
value at the end of the reporting period
d. the bankruptcy of a customer that occurs after the reporting period usually confirms that the
customer was credit-impaired at the end of the reporting period

242. S1. An entity shall prepare its financial statements on a going concern basis if management
determines after the reporting period either that it intends to liquidate the entity or to cease trading,
or that it has no realistic alternative but to do so.
S2. An entity shall disclose the date when the financial statements were authorized for issue and who
gave that authorization.
a. True, True c. False, True
b. True, False d. False, False

243. A statement of financial position as at the beginning of the earliest comparative period should prepare
by an entity in any of the following circumstances, except when an entity
a. applies an accounting policy retrospectively
b. reclassifies items in the financial statements
c. changes in any of its estimates used in accounting
d. makes retrospective restatement of items in the financial statements

244. Significant changes in the market value of trading securities occurring after the balance sheet date
should
a. be treated as a prior period error in next year’s financial statement
b. result in an adjustment of the market value used in the lower of cost or market valuation at
balance sheet date
c. be considered in the valuation of the securities at the balance sheet date and disclosed in the
notes to financial statements
d. not be considered in the valuation of the securities at balance sheet date but disclosed in the
notes to financial statements
245. PAS 10 covers adjusting and nonadjusting events after the reporting period up to
a. date of financial statements publication.
b. date of authorization to issue financial statements.
c. date when financial statements are filed with the regulator.
d. date when financial statements are approved by shareholders.

246. MAE Company had used the weighted method of inventory valuation since it began operations in
2016. The entity decided to change to specific identification method for measuring inventory at the
beginning of 2018.
The following schedule shows year-end inventory balances.
YEAR Weighted Average Specific Identification
2016 3,400,000 4,300,000
2017 6,700,000 6,000,000
2018 7,200,000 6,700,000

What pretax amount should be reported in the statement of retained earnings for 2018 as the
cumulative effect of the change in accounting policy?
a. 500,000 decrease c. 900,000 increase
b. 700,000 decrease d. 1,200,000 decrease

247. Using the information from the preceding number, what pretax amount should be reported in the
statement of retained earnings for 2019 as the cumulative effect of the change in accounting policy,
if the entity decided to change its accounting policy at the beginning of 2019?
a. 500,000 decrease c. 900,000 increase
b. 700,000 decrease d. 1,200,000 decrease

248. Using the information from the preceding number, the following events were reported by MAE
Company during 2019:
 It was decided to write off 1,000,000 from inventory which was over two years old as it was
obsolete.
 Sales invoice No. 0713 had been omitted from the financial statements for the year ended
December 31, 2017, amounting to 500,000
Aside from the change in measuring inventory (weighted average to specific identification), what
amount should be also reported as prior period error in the financial statements for the year ended
December 31, 2019?
a. 1,000,000 decrease c. 500,000 increase
b. 500,000 decrease d. no other amount should be reported

249. Using the information from the preceding number, how much is the net effect of the events
occurred to the retained earnings
a. 1,000,000 decrease c. 500,000 increase
b. 500,000 decrease d. 0

250. MJ Company purchased a machine on January 1, 2015 for 5,000,000. At the date of acquisition, the
machine had a life of five years with no residual value. The machine was depreciated on a straight-
line basis.
On January 1, 2018, the entity determined that the machine had a useful life of seven years from the
date of acquisition.

What is the balance of accumulated depreciation on January 1, 2019?


a. 0 c. 2,500,000
b. 500,000 d. 3,500,000

251. In 2019, a firm changed from the FIFO method of accounting for inventory to weighted average. The
firm’s 2019 and 2018 comparative financial statements will reflect which method or methods?
a. 2018: FIFO, 2019: FIFO c. 2018: Weighted Average, 2019: FIFO
b. 2018: FIFO, 2019: Weighted Average d. 2018: Weighted Average, 2019: Weighted Average
252. In 2019, a firm changed from the straight-line (SL) method of depreciation to double-declining
method (DDB). The firm’s 2019 and 2018 comparative financial statements will reflect which
method or methods?
a. 2018: SL, 2019: SL c. 2018: DDB, 2019: DDB
b. 2018: SL, 2019: DDB d. 2018: SL, 2019: either SL or DDB

253. A company has included in its consolidated financial statement this year a subsidiary acquired
several years ago that was appropriately excluded from consolidated last year. This results in
a. a correction of error.
b. neither an accounting change nor a correction of an error.
c. an accounting change that should be reported prospectively.
d. an accounting change that should be reported by restating the financial statements of all prior
periods presented.

254. The following do not fall within the definition of “related parties” as defined by PAS 24, except
a. Government department and agencies.
b. The wife of a key management personnel who has the authority to plan, direct, and control
the activities of the reporting enterprise.
c. A supplier with whom the reporting entity has a one-year contract for the supply of raw
materials.
d. Providers of finance in the course of their normal dealings with an enterprise by virtue only of
those dealings.

255. According to PAS 24, all of the following are necessarily considered related parties of JMS Company,
except
a. An associate of JMS Company
b. Key management personnel of JMS Company
c. An entity providing banking facilities to JMS Company
d. BS, a shareholder of JMS Company owning 45% of the ordinary share capital

256. Which of the following situations will require disclosures as a related party?
a. In consolidated financial statements in respect to intra-group transactions.
b. In parent, separate financial statements when they are made available or published with
consolidated financial statements.
c. In the financial statements of state-controlled enterprise of transactions with other state-
controlled enterprises.
d. In related party relationships where control exists, irrespective of whether there have been
transactions between related parties.

257. S1. Relationships between a parent and its subsidiaries shall be disclosed only of there are
transactions between them.
S2. An entity shall disclose the name of its parent and, if different, the ultimate controlling party.
a. True, True c. False, True
b. True, False d. False, False

258. S1. Based on PAS 24, items of similar nature may be disclosed in aggregate except when separate
disclosure is necessary for an understanding of the effects of related-party transactions on the
financial statements of the entity.
S2. Disclosures that related-party transactions were made on terms equivalent to those that prevail
in arm’s length transactions are made only if such terms can be sustained.
a. True, True c. False, True
b. True, False d. False, False

259. If an entity publishes a complete set of financial statements in its interim financial report, the form
and content of those statements should conform to
a. PAS 34 c. PAS 1
b. PAS 27 d. PFRS 10

260. The interim financial report is intended to


a. provide an update on the latest complete set of annual financial statements
b. correct error discovered subsequent to the release of the financial statements
c. provide additional information not disclosed previously in the annual financial statements
d. provide information that are requested by the investors in relation to a planned initial public
offering
261. Conceptually, interim financial statements can be described an emphasizing
a. comparability over neutrality c. reliability over understandability
b. relevance over comparability d. timeliness over reliability

262. it is the approach of looking into an entity’s organization and management structure and its interim
financial reporting system to identify the business and geographical segment to external reporting
purposes.
a. Entity approach c. Organizational approach
b. Management approach d. Scientific approach

263. PAS 34 states a presumption that anyone reading interim financial reports will
a. have access to the records of the entity.
b. not make decisions based on the reports.
c. have access to the most recent annual report.
d. understand all Philippine Financial Reporting Standards.

264. JMS Inc. owns a number of farms that harvest produce seasonally. Approximately 80% of the entity’s
sales are in the period August to October. Because the entity’s business is seasonal, PAS 34 suggest
a. no additional disclosure.
b. additional disclosure in the accounting policy note.
c. additional disclosure in the notes about seasonal nature of the business.
d. disclosure of the seasonal nature of the business and disclosure of financial information for the
latest and comparative 12-month period in addition to the interim report.

265. S1. Interim financial report means a financial report containing either a complete set of financial
statements or a set of condensed financial statements for an interim period.
S2. If an entity publishes a complete set of financial statements in its interim financial report, the form
and content of those statements shall conform to the requirements of PAS 1 for a complete set of
financial statements.
S3. An entity shall apply different accounting policies in its interim financial statements and in its
annual financial statements.
a. True, True, True c. True, False, False
b. True, True, False d. False, True, False

266. According to PAS 29, characteristics of the economic environment of a country which indicate the
existence of hyperinflation include:
I. the general population prefers to keep its wealth in non-monetary assets or in a relatively
stable foreign currency. Amounts of local currency held are immediately invested to maintain
purchasing power
II. the general population regards monetary amounts not in terms of the local currency but in
terms of a relatively stable foreign currency. Prices may be quoted in that currency
III. sales and purchases on credit take place at prices that compensate for the expected loss of
purchasing power during the credit period, even if the period is short
IV. interest rates, wages, and prices are linked to a price index
V. the cumulative inflation rate over three years approaches, or exceeds, 100%
a. I or II only c. I, II, III or IV only
b. I, II or III only d. I, II, III, IV or V

267. S1. Intelligent judgment may be used in determining whether restatement of financial statement is
required.
S2. PAS 29 established an absolute rate at which hyperinflation will be deemed to arise.
a. True, True c. False, True
b. True, False d. False, False

268. Given a hyperinflationary economy under PAS 29, which price index is used in restating financial
statements and measuring inflation? (Legend: GPI – General Price Index; CPI – Consumer Price Index)
A. B. C. D.
Restating Financial Statements GPI GPI CPI CPI
Measuring Inflation GPI CPI GPI CPI
269. Money loses purchasing power at such a rate that comparison of amounts from transactions and
events that have occurred at different times, even within the same accounting period is misleading.
This financial accounting problem is addressed through
a. Fair value accounting c. Price-level accounting
b. Peso accounting d. Revaluation of property

270. Under PAS 29, the financial statements of an entity that reports in the currency of a hyperinflationary
economy are stated in terms of
a. Current cost c. lower of cost or market value
b. Historical cost d. measuring unit current at the balance sheet date

271. A company should disclose information about the effects of changing prices in a hyperinflationary
economy
a. in the body of the financial statements
b. in the notes to the financial statements
c. in the management’s report to shareholders
d. as a supplementary information to the financial statements

272. The following statements are based on PAS 29


S1. The financial statements of an entity whose functional current is the currency of a
hyperinflationary economy shall be stated in terms of measuring unit current at the end of the
reporting period (balance sheet date).
S2. The gain or loss on the net monetary position shall be included in profit or loss and separately
disclosed.
S3. When an economy ceases to be hyperinflationary, an entity shall treat the amount expressed in
the measuring unit current at the end of the reporting period as the basis for carrying amounts in the
subsequent financial statements.
a. True, True, True c. True, False, False
b. True, True, False d. False, True, False

273. Upon first time adoption of PFRS, an entity may elect to use fair value as deemed cost for
a. financial liability not held for trading
b. intangible asset for which there is no active market
c. any individual item of property, plant and equipment
d. biological asset related to agricultural activity for which there is no active market

274. Under PFRS 1, how should a first-time adopter of PFRS recognize the adjustments required to present
the opening PFRS statement of financial position?
a. All of the adjustments should be recognized in profit or loss
b. All of the adjustments should be recognized directly in retained earnings or, if appropriate, in
another category of equity.
c. Current adjustments should be recognized in profit or loss and noncurrent adjustments should be
recognized in retained earnings
d. Adjustments that are capital in nature should be recognized in retained earnings and adjustments
that are revenue in nature should be recognized in profit or loss.

275. JMS Corporation is a first-time adopter of PFRS. The most recent financial statements it presented
under previous GAAP were on December 31, 2018. JMS adopted PFRS for the first time and intended
to present the first PFRS financial statements on December 31, 2019. The entity plans to present a
two-year comparative information for years 2018 and 2017. The opening PFRS statement of financial
position should be prepared on
a. January 1, 2017 c. January 1, 2019
b. January 1, 2018 d. January 1, 2020

276. Which is the correct order of the following steps in the accounting cycle?
I. Preparation of Financial Statements
II. Making closing entries in the general journal
III. Posting transaction entries in the general ledger
IV. Making reversing entries in the general journal
a. II, III, IV, I c. III, I, II, IV
b. II, IV, III, I d. III, I, IV, II
277. A voucher system is usually used for transactions involving
a. Cash disbursements c. Cash receipts and disbursements
b. Cash receipts d. Purchase on account

278. The double-entry concept in accounting means which of the following?


a. The debit-credit convention must be used
b. Only two accounts are affected by each transaction recording
c. At least two accounts are affected by each transaction recorded
d. For every asset increased, a revenue or liability must also be increased.

279. Which of the following errors will probably be disclosed by the preparation of a trial balance (would
cause it to be out of balance)?
a. Failure to post part of a journal
b. Failure to post an entire journal entry (nothing is posted)
c. Failure to record an entry in the journal (nothing is entered)
d. Posting the debit of a journal entry as a credit, and the credit as debit

280. The effect of closing entries is to


a. Change assets
b. Change liabilities
c. Change retained earnings
d. Change debit balances of all accounts into credits and vice-versa

281. S1. An entity may be a first-time adopter if, in the preceding year, it prepared PFRS financial
statements for internal management use, as long as those PFRS financial statements were not made
available to owners or external parties such as investors or creditors.
S2. If a set of PFRS financial statements was, for any reason, made available to owners or external
parties in the preceding year, then the entity will be considered to be on PFRSs, and PFRS 1 will be
applied.
a. True, True c. False, True
b. True, False d. False, False

282. An entity can be a first-time adopter if, in the preceding year, its financial statements:
I. Asserted compliance with some but not all PFRSs
II. Compliance with both previous GAAP and PFRSs (Previous GAAP means the GAAP that an
entity followed immediately before adopting to PFRSs.)
III. Compliance with PFRSs even if the auditor's report contained a qualification with respect to
conformity with PFRSs.
IV. Included only a reconciliation of selected figures from previous GAAP to PFRSs.
a. I or II only c. III or IV only
b. II or III only d. I or IV only

283. The following are requirement of PFRS 1 as disclosures that explain how the transition from previous
GAAP to PFRS affected the entity's reported financial position, financial performance and cash flows,
except,
a. reconciliations of equity reported under previous GAAP to equity under PFRS both (a) at the date
of transition to PFRSs and (b) the end of the last annual period reported under the previous GAAP
b. explanation of immaterial adjustments that were made, in adopting IFRSs for the first time, to
the statement of financial position, statement of comprehensive income and statement of cash
flows (the latter if presented under previous GAAP)
c. if the entity recognized or reversed any impairment losses in preparing its opening IFRS statement
of financial position
d. appropriate explanations if the entity has elected to apply any of the specific recognition and
measurement exemptions permitted under PFRS 1 – for instance, if it used fair values as deemed
cost
284. Under PFRS 1, how should a first-time adopter of PFRS recognize the adjustments required to present
opening PFRS statement of financial position?
a. All of adjustment should be recognized in profit or loss.
b. All of the adjustments should be recognized directly in retained earnings or, if appropriate, in
another category of equity.
c. Current adjustments should be recognized in profit or loss and noncurrent adjustments should be
recognized in retained earnings.
d. Adjustments that are capital in nature should be recognized in retained earnings and adjustments
that are revenue in nature should be recognized in profit or loss.

285. Upon first time adoption of PFRS, an entity may elect to use fair value as deemed cost for
a. Financial liability not held trading
b. Intangible asset for which there is no market
c. Any individual item of property, plant and equipment
d. Biological asset related to agricultural activity for which there is no active market

286. An operating segment is a component of an entity:


I. That engages in business activity from which it may earn revenue and incur expenses,
including revenue and expenses relating to transactions with other components of the same
entity
II. Whose operating results are regularly reviewed by the entity’s chief operating decision maker
III. For which discrete financial information is available
IV. May engage in business activity which it has yet to earn revenue
a. I or II only c. I, III or IV only
b. I, II or III only d. I, II, III or IV

287. S1. Segment reporting shall apply to the separate or individual financial of an entity and to the
consolidated financial statements of a group with a parent whose debt or equity instruments are not
traded in a public market.
S2. The purpose of segment reporting is to enable investors and users make better assessment of
each business activity leading to understanding of the position of the entity as a whole.
a. True, True c. False, True
b. True, False d. False, False

288. S1. Chief operating decision maker’s function is to allocate resources to the segments and asses their
performance.
S2. Operating segments are identified based on the components of the entity that are considered to
be important for internal management reporting purposes.
a. True, True c. False, True
b. True, False d. False, False

289. The following are not “quantitative thresholds” of identifying an operating segment, except
I. The segment revenue, including both sales to external customers and intersegment sales or
transfers, is 10% or more of the combined revenue, internal and external, of all operating
segments.
II. The absolute amount of profit or loss is 10% or more of the greater in absolute amount of
combined profit of all operating segments that reported a profit or combined loss of all
operating that reported a loss
III. The assets of the segment are 10% or more of the combined assets of all operating segments
a. I and II only c. I and III only
b. II and III only d. I, II and III

290. The following statements are incorrect concerning the 75% overall size test for reportable segments,
except
I. The total external revenue of all reportable segments is 75% or more of the entity’s external
revenue
II. The total external and internal revenue of all reportable segments is 75% or more of the
entity’s external revenue
III. The total external revenue of all reportable segments is 75% or more of the entity’s external
and internal revenue
IV. The total internal revenue of all reportable segments is 75% or more of the entity’s internal
revenue
a. I only c. I and III only
b. I and II only d. I and IV only
291. S1. Under PFRS 8, there is no precise limit as to number of reportable segments are to be determined
and disclosed.
S2. PFRS 8 suggests that if the number of reportable segments exceeds five but less than ten, it is
likely the information may be too detailed and consequently lose its usefulness
a. True, True c. False, True
b. True, False d. False, False

292. S1. Two or more operating segments may be aggregated even if they have dissimilar nature of product
or service
S2. An entity may combine information about operating segments that do not meet the 75%
thresholds even if the operating segments have dissimilar economic characteristic and share a
majority of the aggregation criteria
a. True, True c. False, True
b. True, False d. False, False

293. According PFRS 8, the entity shall not disclose the following except,
I. General information about the operating segment
II. Information about segment profit or loss, including revenue and expenses included in profit
or loss, segment assets and segment liabilities
III. Reconciliations of the totals of segment revenue, profit or loss, segment assets, segment
liabilities and other material segment items to corresponding items in the entity’s financial
statements.
a. I only c. I and III only
b. I and II only d. I, II and III

294. S1. Under PFRS 8, an entity disclosed factors used to identify reportable segments including the basis
of organization is a general information
S2. Types of products and services from which each reportable segment derives its revenue should be
disclosed as information about segment profit or loss according to PFRS 8.
a. True, True c. False, True
b. True, False d. False, False

295. Under PFRS 8, the entity shall provide all of the following reconciliations, except
a. The total for all item of information, whether material or not, disclosed by the reportable
segments to the corresponding amount for the entity
b. The total assets and liabilities of all reportable segments to the entity’s respective total assets and
liabilities
c. The total revenue for all reportable segments to the entity revenue
d. The total profit or loss of all reportable segments to the entity profit or loss before income tax
expense and discontinued operations

296. Entity-wide disclosures include all, except


a. Information about major customers
b. Information about intersegment sales
c. Information about products and services
d. Information about geographical areas

297. Under PFRS 8, an entity shall disclose the following geographical information, except,
a. Revenue from external customers in the entity’s country of domicile, and in all foreign operations
in total
b. Current assets located in the entity’s country of domicile and in all foreign countries in total
c. Separate disclosures of material revenue from external customers in an individual foreign country
d. The basis for attributing revenue from external customers to individual countries.

298. S1. A major customer is defined as a single internal customer providing revenue which amounts to
10% or more of an entity’s internal revenue
S2. The major customer disclosure means that an entity shall provide information about the extent of
reliance on the major customers
a. True, True c. False, True
b. True, False d. False, False
299. The following statements are true with respect to a chief operating decision maker, which is false?
a. The term chief operating decision maker identifies a function and not necessarily a manager with
a specific title
b. The board of directors acting collectively could qualify as the chief operating decision maker
c. The chief internal auditor who reports to the board of directors usually plays a very important
role and would generally qualify as chief operating decision maker
d. In some cases, the chief operating decision maker could be the chief operating officer

300. An entity shall disclose each reportable segment all of the following specified amounts included in the
measure of profit or loss, except
a. Depreciation and amortization
b. General corporate expenses
c. Income tax expense
d. The entity’s interest in the profit or loss of associated and joint venture accounted for by the
equity method

301. The following does not constitute the definition of discontinued operations, except
I. Component of an entity that is imaginable to be disposed or classified as held for sale
II. Represents a separate major line of business or geographical area of operations
III. Part of a single coordinated plan to dispose of a separate major line of business or
geographical area of operations
IV. A subsidiary acquired exclusively with a view to resale
a. I, II and III c. I, III and IV
b. II, III and IV d. I, II and IV

302. S1. A component of an entity is classified as discontinued operation at the date of entity has actually
disposed of the operation, or when the operation meets the criteria to be classified as held for sale.
S2. Prospective classification is allowed by PFRS 5 for a discontinued operation that met the
discontinued criteria after the end of the reporting period.
a. True, True c. False, True
b. True, False d. False, False

303. S1. If the discontinued criteria are met after the end of the reporting period, the entity can classify
the discontinued operation as held for sale in the current financial statements.
S2. Results of discontinued operation shall be shown, gross of tax, as either single or multiple amount
in the income statement after the income from continuing operations.
a. True, True c. False, True
b. True, False d. False, False

304. S1. The assets of the component classified as held for sale shall be presented as a single amount under
current liabilities and the liabilities of the component shall be presented as a single amount under
current assets.
S2. Noncurrent asset of the component held for sale shall be continued to be depreciated.
a. True, True c. False, True
b. True, False d. False, False

305. In preparing comparative financial statements, if a disposal group is classified as held for sale in the
current year, how will the asset and liabilities of the disposal group be presented?
a. Reclassify or represent asset and liabilities of prior year (retrospective); reclassify or represent
asset and liabilities of current year
b. Reclassify or represent asset and liabilities of prior year (retrospective); shall not reclassify nor
represent asset and liabilities of current year
c. Shall not reclassify nor represent asset and liabilities of prior year; reclassify or represent asset
and liabilities of current year (prospective)
d. Shall not reclassify nor represent asset and liabilities of prior year; shall not reclassify nor
represent asset and liabilities of current year
306. An entity manufactures and sells household products. The entity experienced losses associated with
the small appliance group. Operations and cash flows for this group can be clearly distinguished from
the rest of the entity’s operations. The entity decided to sell the small appliance group. The following
are the latest point at which the entity shall report the small appliance group as a discontinued
operation, except
I. When the entity classifies it as held for sale
II. When the entity receives an offer for the segment
III. When the entity first sells any of the assets of the segment
IV. When the entity sells the majority of the assets of the segment.
a. I only c. II and III only
b. I and II only d. IV only

307. The following are not requirement for a component of an entity to be classified as a discontinued
operation, except
I. The activities must cease permanently prior to the financial statements being authorized for
issue
II. The component must be a reportable segment
III. The assets must have been classified as held for sale in the previous financial statements
IV. The component must have been a cash generating unit while being held for use
a. I only c. II and III only
b. I and II only d. IV only

308. The following criteria is not required for the results of a component of an entity to be classified as
discontinued operations, except
I. Management must have entered into a sale agreement.
II. The component is available for immediate sale.
III. The operations and cash flows of the component shall be eliminated from the operations of
the entity as a result of the disposal
IV. The entity shall not have any significant continuing involvement in the operation of the
component after disposal
a. I only c. I, III and IV
b. II, III and IV d. IV only

309. What is the best evidence an asset’s fair value less cost to sell?
a. The carrying value of the asset
b. The fair value in an active market
c. The selling price in a binding agreement
d. The best estimate of knowledgeable parties

310. An entity shall measure a noncurrent asset or disposal group classified as ‘held for sale’ at
a. Carrying amount
b. Fair value less cost to sell
c. Carrying amount or ‘fair value less cost to sell’, whichever is lower
d. Carrying amount or ‘fair value less cost to sell’, whichever is higher

311. S1. Inventories are assets which are held for sale in the ordinary course of business, in the process of
production for such sale or in the form of materials or supplies to be consume in the production
process or in the rendering of services.
S2. Only items in the warehouse of the entity/owner are inventoriable according to PAS 2.
a. True, True c. False, True
b. True, False d. False, False

312. S1. FOB destination means that the ownership of the goods purchased is vested in the buyer upon
the shipment thereof.
S2. A seller who delivers the goods free alongside bears all expenses and risk of loss until the goods
are unloaded at which time title and risk of loss shall pass to the buyer.
a. True, True c. False, True
b. True, False d. False, False
313. S1. Under perpetual inventory system, the cost of goods sold is computed only at the end of the period
by deducting the physical inventory from the total cost of goods available for sale.
S2. When periodic inventory system is used, a physical count of the units on hand should at least be
made once a year or frequent intervals to confirm the balances appearing on the stock cards.
a. True, True c. False, True
b. True, False d. False, False

314. S1. Trade discounts are reductions in the invoice price allowed only when payment is made within the
discount group.
S2. Cash discounts are reductions in the list price or catalog price in order to get the invoice price or
the amount actually charged to the buyer.
a. True, True c. False, True
b. True, False d. False, False

315. The following are done when using gross method of recording purchases, except
I. The cost of purchases is measured after deducting cash discounts allowable whether taken or
not taken.
II. Cash discounts taken are recorded in a purchases discount account at the time of payment
III. Purchases discount is deducted from purchases when measuring cost of goods sold.
a. I only c. III only
b. II only d. I and III only

316. The following comprises the cost of purchase of an inventory, except


I. Trade discounts
II. Purchase price
III. Rebates
IV. Import duty and irrecoverable tax
V. Cost of conversion
a. I only c. V only
b. I and III only d. III and IV only

317. The following are included from the cost of inventory and recognized as expenses in the period when
incurred, except
I. Storage cost related to goods in process or part-finished goods
II. Abnormal amounts of wasted materials, labor and other production costs
III. Administrative overheads that do not contribute to bringing inventories to their present
location and condition
IV. Distribution costs
a. I only c. II and III only
b. I and II only d. IV only

318. The cost of inventory of a service provider comprises of the following, except
I. Labor of personnel directly engaged in providing the service
II. Compensation of supervisor directly engaged in providing the service
III. Administrative compensation of those dealing with clerical administrative duties
IV. Attributable overhead incurred in providing the service
a. I only c. I and II only
b. III only d. III and IV only

319. The following should not be reported as inventory, except


I. Land acquire for resale by a real estate dealer
II. Shares and bonds held for resale by a brokerage firm
III. Partially completed goods held by manufacturing entity
IV. Machinery acquired by a manufacturing entity for use in the production process
a. I, II and III c. I, II and IV
b. II, III and IV d. I, III and IV
320. A manufacturer sells merchandise to a retailer which in turn sells the goods to the public. The retailer
purchases from the manufacturer under a consignment contract. The following event will not
constitute the recognition of revenue by the manufacturer:
I. When goods are sold by the retailer
II. When goods are delivered to the retailer.
III. The recognition depends on the terms of delivery
IV. The recognition depends on the terms of payment
a. I, II and III c. I, II and IV
b. II, III and IV d. I, III and IV

321. Interim Reporting. On June 30, 2018, MAE Company incurred a P 100,000 net loss from disposal of a
business segment. Also, on June 30, 2018, the entity paid P 40,000 for property taxes assessed for the
year 2018. What amount should be included in the determination of net income or loss for the six-
month interim period ended June 30, 2018?
A. P 140,000 C. P 90,000
B. P 120,000 D. P 70,000

322. Interim Reporting. JM Company reported P 4,750,000 net income for the quarter ended September
30, 2018 which included the following after tax items:
 A P 3,000,000 expropriation gain, realized on April 30, 2018, was allocated equally to the
second, third, and fourth quarters of 2018.
 An P 800,000 cumulative-effect loss resulting from a change in inventory valuation method,
from FIFO to Weighted Average, was recognized on August 1, 2018
The entity paid P 2,400,000 on February 1, 2018, for 2018 calendar year property taxes. Of this amount
P 600,000 was allocated to the third quarter ended September 30, 2018. What amount should be
reported as net income for the third quarter?
A. P 4,550,000 C. P 5,550,000
B. P 5,150,000 D. P 5,750,000

323. Hyperinflationary Environment. JB Company reported the following machinery on December 31,
2018:
Cost Accumulated Depreciation
Acquired in December 2015 P 4,000,000 P 1,600,000
Acquired in December 2017 1,000,000 200,000

Index numbers at the end of each year are 120 for 2015, 122 for 2016, 125 for 2017 and 350 for 2018.
What should be reported in a hyperinflationary statement of financial position prepared on December
31, 2018 as the carrying amount of the machinery?
A. P 3,200,000 C. P 8,960,000
B. P 7,800,000 D. P 9,240,000

324. Operating Segment. MJ Company had no intersegment sales and provided the following data for the
current year (in thousands):
Segment Revenue Profit (Loss) Assets
1 620 200 400
2 100 20 80
3 340 70 300
4 190 (30) 140
5 180 (25) 180
6 70 10 120
7 120 (20) 140
Others 380 (25) 140
 The “others” category includes five operating segments, none of which has revenue or assets
greater than P 80,000 and none with an operating profit
 Operating Segments 1 and 2 produce similar products and use very similar production processes,
but serve differently customer types and use quite different product distribution system
These differences are due in part to the fact that Segment 2 operates in a regulated environment
while Segment 1 does not.
 Operating Segments 6 and & have very similar products, production processes, product
distribution systems, but are organized as separate divisions since they are substantially different
types of customers.
Neither Segments 6 and 7 operate in a regulated environment.
What are the reportable segments for the current year?
A. Segments 1, 3, 4 and 5
B. Segments 1, 3, 4, 5 and 7
C. Segments 1, 2, 3, 4 and 5
D. Segments 1, 3, 4, 5, and Segments 6 and 7 combined as one segment

325. Operating Segment. JBM Company, a publicly owned entity, assesses performance and makes
operating decisions using the following information for the reportable segments:
Total Revenue P 7,700,000
Total Profit 500,000
The profit included intersegment profit of P 50,000. In addition, the entity has P 10,000 of common
cost for the reportable segments that are not allocated in reports reviewed by the chief operating
decision maker.
What amount should be reported as segment profit?
A. P 550,000 C. P 500,000
B. P 510,000 D. P 450,000

326. Operating Segment. ED Company has three lines of business, each of which was determined to be
reportable segment.
The entity sales aggregated P 7,500,000 in the current year, of which Segment No. 1 contributed 40%.
Traceable costs were P 1,750,000 for Segment No. 1 out of a total of P 5,000,000 for the entity as a
whole.
For external reporting, the entity allocated common costs of P 1,500,000 based on the ratio of a
segment’s income before common costs to the total income before common costs.
In the financial statements for the current year, what amount should be reported as profit for
Segment No. 1?
A. P 1,250,000 C. P 650,000
B. P 1,000,000 D. P 500,000

327. Discontinued Operations. Jay Company operates two restaurants, one in Potipot and one in Lingayen.
The operations and cash flows of each two restaurants are clearly distinguishable.
During 2018, the entity decided to close the restaurant in Lingayen and sell the property. It is probable
that the disposal will be completed early next year.
The revenue and expenses for 2018 and for the preceding two years are as follows (in pesos):
2018 2017 2016
Sales – Potipot 60,000 48,000 40,000
Cost of goods sold – Potipot 26,000 22,000 18,000
Other expenses – Potipot 14,000 13,000 12,000
Sales – Lingayen 23,000 30,000 52,000
Cost of goods sold – Lingayen 14,000 19,000 20,000
Other expenses – Lingayen 17,000 16,000 15,000
The other expenses do not include income tax expense. During the later part of 2018, the entity sold
some of the kitchen equipment of the Lingayen restaurant and recognized pretax gain of P 15,000 on
the disposal.
What amount should be reported as pretax income or loss from discontinued operation for 2018?
A. P 8,000 loss C. P 5,600 loss
B. P 7,000 gain D. P 1,000 gain

328. Discontinued Operations. On December 31, 2018, MJ Company committed to a plan to discontinue
the operations of Lubricant Division.
The fair value of the facilities was P 1,000,000 less than carrying amount on December 31, 2018.
The division’s operating loss for 2018 was P 2,000,000 and the division was actually sold for P
1,200,000 less than the carrying amount in 2019.
The entity estimated that the division operating loss for 2019 would be P 500,000.
What amount should be reported as pretax loss from discontinued operation 2018?
A. P 1,000,000 C. P 3,000,000
B. P 2,000,000 D. P 3,200,000

329. Discontinued Operations. JAYMAE Company had two operating divisions, one manufactures
machinery and the other breeds and sells horses. Both divisions are considered separate components.
The horse division has been unprofitable and on November 15, 2018 the entity adopted as formal
plan to sell the division. At December 31, 2018 the component was considered for sale.
The sale was completed on April 30, 2019.
On December 31, 2018, the carrying amount of the assets of the horse division was P 5,000,000. On
that date, the fair value of the assets less cost of disposal was P 4,000,000.
The before-tax operating loss of the division for the year was P 2,000,000.
The after-tax tax income from continuing operations for 2018 was P 8,000,000. The income tax rate
is 30%.
What is the net income for 2018?
A. P 3,850,000 C. P 5,600,000
B. P 4,500,000 D. P 6,250,000

330. Discontinued Operations. EMJAY Company had two operating divisions, one manufacturing farm
equipment and the other office supplies. Both divisions are considered separate components.
The farm equipment component had been unprofitable and on September 1, 2018, the entity adopted
a plan to sell the assets of the division.
The actual sale was effected on December 15, 2018 at a price of P 3,000,000. The carrying amount of
the division’s assets was P 5,000,000.
The division incurred before-tax operating loss of P 1,500,000 from the beginning of the year through
December 15, 2018.
The entity’s after-tax income from continuing operations is P 9,000,000.
The income tax rate is 30%.
What amount should be reported as net income for the current year?
A. P 5,500,000 C. P 6,550,000
B. P 6,300,000 D. P 7,600,000

331. Inventory – Error. In reviewing the draft financial statements for the year ended December 31, 2019,
TOPAK Company decided that market conditions were such that the provision for inventory
obsolescence on December 31, 2019 should be increased by P 3,000,000.
If the same basis of calculating inventory obsolescence had been applied on December 31, 2018, the
provision would have been P 1,800,000 higher than the amount recognized in the statement of
comprehensive income.
What amount should be made to the net income of 2019?
A. P 3,000,000 decrease C. P 1,200,000 decrease
B. P 3,000,000 increase D. P 1,200,000 increase

332. Using the information from the preceding number, what adjustment should be made to the net
income of 2018 presented as comparative figure in the 2019 financial statements?
A. P 3,000,000 increase C. P 1,200,000 increase
B. P 1,800,000 decrease D. 0

333. Prior Period Error. DPS Company reported the following events during the year ended December 31,
2019:
 A counting error relating to the inventory on December 31, 2018 was discovered. This required
a reduction in the carrying amount of inventory at that date of P 2,000,000.
 The provision for uncollectible accounts receivable on December 31, 2018 was P 500,000. During
2019, P 800,000 was written off related to the December 31, 2018 accounts receivable.
 The income tax rate is 30%.
What adjustment is required to restate retained earnings on January 1, 2019?
A. P 2,500,000 C. P 1,400,000
B. P 2,000,000 D. 0

334. Inventory – Basic Problem. JMS Company included the following in inventory at year-end:
Merchandise out on consignment at sale price, including 30% 1,500,000
markup on sales
Goods purchased in transit, shipped FOB shipping point 1,200,000
Goods held on consignment by JMS 900,000
At what amount should the inventory be reduced?
A. P 1,350,000 C. P 2,400,000
B. P 2,100,000 D. P 3,600,000
335. Inventory – Basic Problem. On August 1, LOVE Company recorded purchases of inventory of P 800,000
and P 1,000,000 under credit terms of 2/15, net 30.
The payment due on the P 800,000 purchase was remitted on August 16. The payment due on the P
1,000,000 purchase was remitted on August 31.
Under the net method and the gross method, these purchases should be included at what respective
amounts in the determination of cost of goods available for sale?
Net Method Gross Method
A. P 1,784,000 P 1,764,000
B. P 1,764,000 P 1,800,000
C. P 1,764,000 P 1,784,000
D. P 1,800,000 P 1,764,000

336. Which of the following is not true about accounting for inventory?
A. FIFO is allowed.
B. Interest costs should not be capitalized.
C. The weighted-average method is acceptable.
D. Inventories are always valued at net realizable value.

337. During the year, an entity transferred inventory to another entity and agreed to repurchase the
merchandise early in next year. The transferee used the inventory as collateral to borrow from a bank,
remitting the proceeds to the transferor. When the transferor repurchased the inventory, the
transferee used the proceeds to repay the bank loan. This transaction is known as
A. Assignment for the benefit of creditors. C. Installment sale.
B. Consignment. D. Product financing arrangement.

338. Which is not a common disclosure for inventories


A. Inventory composition C. Inventory financing arrangement
B. Inventory cost method D. Inventory location

339. If an item of inventory that was written down to net realizable value in a prior period subsequently
recovers, then
A. Previous amount of the write-down can be reversed.
B. Carrying amount of the inventory cannot be adjusted.
C. Value adjustment can be recognized immediately in equity.
D. Adjustment must be recognized in a “provision for future inventory write-downs” account

340. The specific identification method can be used only


A. In income tax returns
B. For financial reporting purpose but not in income tax returns
C. When the individual items in inventory is similar in terms of cost, function, and sales value
D. When the actual cost of individual units can be determined from the accounting records.

341. Generally accepted accounting principles require the selection of an inventory cost flow method
which
A. Most clearly reflects the periodic income
B. Yield the most conservative amount of reported income
C. Matches the physical flow of goods for inventory with sales revenue
D. Most closely approximates lower of cost and net realizable value for ending inventory.

342. Under PAS 2, the fixed production overhead is allocated to the inventory units on the basis of
production facility’s
A. Actual capacity C. Normal capacity
B. Ideal capacity D. Theoretical capacity

343. Which of the following statements is incorrect regarding LCNRV?


A. In most situations, entities price inventory on a total inventory basis
B. Entities can use an allowance account in reducing inventory at NRV
C. NRV is estimated selling price less estimate cost to complete and cost to make a sale
D. One of two methods may be used to record the income effect of valuating inventory at LCNRV
344. In a perpetual inventory system, recording a sale on account involves debiting which of the following
accounts?
A. Only Accounts Receivable.
B. Accounts Receivable and Inventory.
C. Accounts Receivable and Cost of Goods Sold.
D. Accounts Receivable, Cost of Goods Sold, and Inventory.

345. In manufacturing company, the just-in-time concept of inventory management is best illustrated by
A. Setting finished products before they go out of style.
B. An automated factory with reduced production time below that of other companies in the
industry.
C. Completing the manufacturing process just before the deadline established by the customer.
D. Receiving deliveries of materials from supplies just before materials are used in the production
process.

346. The cost of land typically includes the purchase price and all of the following costs, except
A. Private driveways and parking lots.
B. Grading, filing, draining, and clearing costs.
C. Street lights, sewers, and drainage systems cost.
D. Assumption of any liens or mortgages on the property.

347. The cost of demolition of unwanted building purchased as part of a parcel of land shall be charged to
A. Profit or loss
B. Profit or loss (preferred treatment); cost of land (alternative treatment)
C. Cost of land (preferred treatment); cost of new building (alternative treatment)
D. Cost of new building (preferred treatment); cost of land (alternative treatment)

348. Discounts given for early payment of credit purchases of operational assets should be
A. Recorded as interest revenue at purchase date.
B. Recorded as interest expense at purchase date.
C. Deducted from the invoice price when determining the cost of the asset.
D. Capitalized as cost of the asset acquired and subsequently allocated to depreciation expense.

349. When payment for item of property, plant and equipment is deferred beyond normal credit terms,
the difference between the cash price and total payments is
A. charged to retained earnings
B. interest expense over the credit period
C. capitalized as Property, Plant and Equipment
D. interest expense over the useful life of the asset

350. A lessee incurred cost to construct office space in a leased warehouse. The estimated useful life of
the office is ten years. The remaining term of the renewable lease is fifteen years. The cost should be
A. Expensed as incurred
B. Capitalized as leasehold improvements and depreciated over ten years
C. Capitalized as leasehold improvements and depreciated over fifteen years
D. Capitalized as leasehold improvements and expensed in the year in which the lease expires

351. A newly acquired plant asset is to be depreciated over its useful life. The rationale for this process is
the
A. economic entity assumption C. materiality assumption
B. going concern assumption D. monetary unit assumption

352. An entity installed a new production facility and incurred a number of expenses at the point of
installation. The entity’s accountant is arguing that most expenses do not qualify for capitalization.
Included in those expenses are initial operating losses. Those should be
A. Deferred and amortized over a reasonable period of time
B. Expensed and charged to profit or loss in the income statement
C. Capitalized as part of the cost of the plant as a directly attributable cost
D. Taken to retained earnings since it is unreasonable to treat it as part of profit or loss.
353. Accumulated depreciation, as used in accounting, represents
A. Funds set aside to replace assets
B. An expense on the income statement
C. The portion of the asset cost written off as an expense since the acquisition date
D. Earning retained in the business that will be used to purchase another operational asset when the
related asset becomes fully depreciated.

354. The following statements are based on PAS 16 (Property, Plant and Equipment):
S1. An entity required to measure the residual value of an item of property, plant and equipment as
the amount it estimates it would receive current for the asset if the asset were already of the age and
in the condition expected at the end of its useful life.
S2. An entity required to determine the depreciation charged separately for each significant part of
an item of property, plant and equipment.
S3. If fair value can be measure reliably, an entity should carry all items of property, plant and
equipment of a class at a revalued amount, which is the fair value of the items at the date of the
revaluation less any subsequent depreciation and accumulated impairment losses.
A. True, True, True C. True, False, False
B. True, True, False D. False, True, True

355. Normally, depreciation should not be recognized on plant assets during their construction period.
What is the exception?
A. When the length of the construction period is unduly prolonged
B. When the carrying amount of the property cannot be realized through sale
C. When evidence indicates that operations will not produce sufficient revenue to cover all costs,
including depreciation.
D. When partial use of the asset can be identified with an income during their construction period
and corresponding costs can be ascertained.

356. In the case of grants related to an asset, which of these treatments is prescribed by PAS 20?
A. Take it the income statement and disclose it as an extraordinary gain
B. Record the grant at a nominal value in the first year write it off in the subsequent year
C. Record the grant at fair value in the first year and take it to income in the subsequent year
D. Either set up the grant as deferred income or deduct it in arriving at the carrying amount of the
asset.

357. Under PAS 20, a government grant shall be recognized when


A. The entity has complied with all the conditions attaching to the grant.
B. The entity has complied with all the conditions attaching to the grant and the grant will be
received.
C. There is a reasonable assurance that the entity will comply with the conditions attaching to the
government grant.
D. There is a reasonable assurance that the entity will comply with the conditions attaching to the
grant and the grant will be received.

358. Which can be considered a government assistance?


A. Imposition of trading constraints on competitors.
B. Improved facilities, such as irrigation or water supply.
C. Free technical or marketing advice and provision of guarantee.
D. Infrastructure by improvement to the general transport and communication network.

359. Which of the following statements is incorrect in relation to government grant?


A. In respect of loan from the government at zero interest rate, an imputed interest charge should
be made in profit or loss.
B. Any adjustment needed when government grant becomes repayable is accounted for as a change
in accounting estimate.
C. Where conditions apply to a government grant, it should only be recognized when there is
reasonable assurance that the conditions will be met.
D. A government grant that becomes receivable as compensation for losses already incurred should
be recognized as income of the period in which it becomes receivable.

360. The following are based on PAS 20


S1. Government grants shall be recognized in profit or loss on a systematic basis over the periods in
which the entity recognizes as expense the related costs for which the grants are intended to
compensate.
S2. A government grant that becomes receivable as compensation for expenses or losses already
incurred or for the purpose of giving immediate financial support to the entity with no future related
costs shall be recognized in profit or loss of the period in which it becomes receivable.
A. True, True C. False, True
B. True, False D. False, False

361. Which of the following is true regarding the income approach for government grant?
A. Depreciation is higher if the grant is an adjustment of the asset
B. Depreciation is higher and net income lower if the grant is an adjustment to the asset
C. Depreciation is higher if the grant is a deferred revenue and net income is not affected
D. Depreciation is higher and net income lower if the grant is recorded as deferred revenue

362. If the cost of the asset is recorded net of government grant


A. Asset will likely be understated C. Liability will likely be overstated
B. Equity will likely be overstated D. Net income will likely be understated

363. In case of nonmonetary grant, which of the following is prescribed by PAS 20?
A. Record the grant at a value estimated by the management
B. Record the asset at replacement cost and the grant in a nominal value
C. Both the grant and the asset fair value of the nonmonetary asset
D. Record only the asset fair value and not recognize the fair value of the grant

364. PAS 23 defines qualifying assets as assets that necessarily take a substantial period of time to get it
ready for its intended use or sale. Which of the following is not a qualifying asset?
A. Inventories such as wine and cigars
B. Building that will take three years to construct
C. Manufacturing plant and power generation facilities
D. Machinery that is purchased under three-year installment method

365. Borrowing costs related to a qualifying asset shall be


A. Capitalized.
B. Expensed in the period incurred.
C. Neither capitalized nor expensed in the period incurred.
D. Capitalized or expensed in the period incurred, whichever is more convenient.

366. It is permissible to capitalize interest on


A. Assets under construction
B. Inventories routinely manufactured in large quantities and repetitive basis.
C. Assets that already are in use or are ready for their intended use in the earning activities on the
entity.
D. Assets that are not being used in earning activities of the entity and that are not undergoing the
activities necessary to get them ready for such use

367. A company constructed machinery for its own use. A bank loan specifically financed this property both
during and after the construction. How much of the interest should be recorded as interest expense?
A. All interest incurred. C. Interest incurred before completion .
B. Interest incurred after completion. D. Zero.

368. Under PAS 23, Borrowing Costs, which of the following statements about capitalization of borrowing
costs as part of a qualifying asset is true?
A. Capitalization always continues as soon expenditure of the asset is incurred.
B. Capitalization always commences as soon as expenditure of the is incurred.
C. Capitalization always commences as soon as interest on relevant borrowings is being incurred.
D. If funds come from general borrowings, the amount to be capitalized is based on the weighted-
average cost of borrowing.

369. Capitalization of construction period interest is based primarily upon the


A. comparability principle. C. matching principle.
B. full-disclosure principle. D. revenue principle.
370. Interest cost that is capitalized should
A. be written off over the remaining term of the debt.
B. not be written off until the related asset is fully depreciated or disposed of.
C. be accumulated in a deferred charge account and written off equally over a 40-year period.
D. None of these.
371. Which of the following statements best describes the term “significant influence”?
A. The mutual sharing in the risk and benefits of a combined entity.
B. The contractually agree sharing of control over an economic entity.
C. The holding of a significant proportion of the share capital in another entity.
D. The power to participate in the financial and operating policy decisions of an entity.

372. When a company holds between 20% and 50% of the outstanding stock of an investee, which of the
following statements applies?
A. The investor should always use the equity method to account for its investment.
B. The investor should always use the fair value method to account for its investment.
C. The investor must use the fair value method unless it can clearly demonstrate the ability to
exercise “significant influence” over the investee.
D. The investor should use the equity method to account for its investment unless circumstances
indicate that it is unable to exercise “significant influence” over the investee.

373. Which is an incorrect application of equity method for an investment in associate?


A. Investment is recognized at cost on initial recognition
B. The investor’s share in the associate’s profit is included in the investor’s profit or loss
C. Distributions received from an associate increases the carrying amount of the investment
D. The carrying amount is decreased by the investor’s share in the loss of the associate after the date
of acquisition

374. PAS 28 does not require the equity method to be applied by an associate acquired and held with a
view to its disposal within a certain time period. Per PFRS 5, what is the time period within which the
associate must be disposed of?
A. Six months C. Two years
B. Twelve months D. In the near future

375. The following statements are based on PAS 28 (Investment in Associates)


S1. An investment in an associate shall be accounted for using the equity method (benchmark) or cost
method (alternative).
S2. An investor shall discontinue the use of equity method from the date when it ceases to have
significant influence over an associate and shall account on the loss of significant influence.
S3. On the loss of significant influence, the investor shall measure at historical cost of any investment
the investor retains in the former associate.
A. False, True, True C. False, False, True
B. False, True, False D. False, False, False

376. Cistine Company provided the following information at the end of current year.
Finished goods in storeroom, at cost, including overhead of P 400,000 or 2,000,000
20%
Finished goods in transit, including freight charge of P 20,000, FOB shipping 250,000
point
Finished goods held by salesmen, at selling price, cost, P 100,000 140,000
Goods in process, at cost of materials and direct labor 720,000
Materials 1,000,000
Materials in transit, FOB destination 50,000
Defective materials returned to suppliers 100,000
Shipping supplies 20,000
Gasoline and oil for testing finished goods 110,000
Machine lubricants 60,000
What is the correct amount of inventory?
A. 4,000,000 C. 4,170,000
B. 4,090,000 D. 4,270,000
377. Brilliantine Company has incurred the following costs during the current year:
Cost of purchases based on vendors’ invoices 5,000,000
Trade discounts on purchases already deducted from vendors’ invoices 500,000
Import duties 400,000
Freight and insurance on purchases 1,000,000
Other handling costs relating to imports 100,000
Salaries of accounting department 600,000
Brokerage commission paid to agents for arranging imports 200,000
Sales commission paid to sales agents 300,000
After-sales warranty costs 250,000
What is the total cost of purchases?
A. 6,700,000 C. 6,100,000
B. 6,500,000 D. 5,700,000

378. Towertin Company made the following acquisitions during the year:
 Purchased for P 5,400,000, including appraiser fee of P 50,000, a warehouse building and the
land on which it is located.
The land had an appraiser value of P 2,000,000 and original cost of P 1,400,000. The building
had an appraised value of P 3,000,000 and original cost of P 2,800,000.
 Purchased an office building and the land on which it is located for P 7,500,000 cash and
assumed an existing P 2,500,000 mortgage.
For realty tax purposes, the property is assessed at P 9,600,000, 60% of which is allocated to
the building.
What is the total cost for land?
A. 5,000,000 C. 6,000,000
B. 5,840,000 D. 6,160,000

379. Using the information given in the preceding number, what is the total cost of building?
A. 9,240,000 C. 8,760,000
B. 9,000,000 D. 7,760,000

380. On January 1, 2018, Histine Company received a grant of P 1,500,000 from the government to
subsidize tuition fees for a period of 5 years.
On January 1, 2020, the entity violated certain conditions attached to the grant, and therefore had to
repay fully such grant to the government.
What is the grant income for 2018?
A. 1,500,000 C. 300,000
B. 600,000 D. 0

381. Using the information given in the preceding number, what amount should be recognized as loss
resulting from the repayment of the grant in 2020?
A. 1,500,000 C. 300,000
B. 600,000 D. 0

382. During 2018, Istine Company constructed asset costing P 5,000,000. The weighted average
expenditures totaled P 3,000,000.
To help pay for construction, P 2,200,000 was borrowed at 10% January 1, 2018.
Funds not needed for construction were temporarily invested in short-term securities yielding P
45,000 in interest revenue.
Other than the construction funds borrowed, the only other debt outstanding during the year was a
P 2,500,000. 10-year, 9% note payable dated January 1, 2017.
What amount of interest should be capitalized during 2018?
A. 472,000 C. 247,000
B. 300,000 D. 150,000

383. Using the information given in the preceding number, what amount should be reported as interest
expense for 2018?
A. 225,000 C. 153,000
B. 178,000 D. 0
384. At the beginning of current year, Sistine Company acquired 20% of the outstanding ordinary shares of
B Company for P 8,000,000. The carrying amount of the acquired shares was P 6,000,000.
The excess of cost over carrying amount was attributed to a depreciable asset which was undervalued
on B’s statement of financial position and which had a remaining useful life of ten years.
The investee reported net income of P 1,800,000 and paid cash dividends of P 400.000 and thereafter
issued 5% share dividend during the current year.
What amount should be reported as investment income for the current year?
A. 360,000 C. 240,000
B. 340,000 D. 160,000

385. Using the information given in the preceding number, what is the carrying amount of the investment
in associate at year-end?
A. 8,080,000 C. 7,800,000
B. 8,000,000 D. 7,720,000

386. When an investment ceases to be an associate and is accounted for in accordance with IFRS 9, the fair
value of the investment at the date when it ceases to be an associate’
A. Is regarded as its cost on initial recognition as a financial asset.
B. Is regarded as its fair value on initial recognition as a financial asset.
C. Is regarded as its fair value on initial recognition as a financial liability.
D. Is regarded as its amortized cost on initial recognition as an investment.

387. How is the impairment test carried out for an investment in associate?
A. The goodwill is impairment tested individually.
B. The entire carrying amount of the investment is tested for impairment by comparing the
recoverable amount with carrying amount.
C. The carrying amount of the investment shall be compared with fair value.
D. The recoverable amounts of all investments in associates shall be assessed together.

388. An investor uses the equity method to account for investment in ordinary shares. The purchase price
implies a fair value of the investee’s depreciable assets in excess of the investee’s net asset carrying
amount. The investor’s amortization of the excess
A. Decreases the investment account C. Increases the investment income account
B. Decreases the goodwill account D. Does not affect the investment account

389. If an associate has outstanding cumulative preference shares held by outside interests, the investor
computes share of profit or loss
A. After adjusting for preference dividends which were actually paid during the year.
B. Without regard for preference dividends.
C. After adjusting for the preference dividends only when declared.
D. After adjusting for the preference dividends, whether or not the dividends have been declared.

390. An investor uses the equity method of accounting for a thirty percent ownership in an investee. At
year-end, the investor has a receivable from the investee. How should the receivable be reported in
the investor’s financial statements for the current year?
A. None of the receivable should be reported but the entire receivable should be offset against the
investee’s payable to the investor.
B. Seventy percent of the receivable should be separately reported with the balance offset against
the thirty percent of the investee’s payable to the investor.
C. The total receivable should be disclosed separately.
D. The total receivable should be included as part of the investment in associate, without separate
disclosure.

391. S1. There is an impairment if the recoverable amount is higher than carrying amount.
S2. The impairment loss is recognized in profit or loss and presented separately in the income
statement.
A. True, True C. False, True
B. True, False D. False, False
392. PFRS 13, paragraph 72, enumerates the following fair value hierarchy or best evidence of fair value:
I. The unobservable inputs for the asset that are usually developed by the entity using the best
available information from the entity’s own data.
II. The quoted prices for similar assets in an active market and quoted prices for identical or
similar assets in a market that is not active.
III. The quoted prices in an active market for identical assets.
How should the aforementioned fair value hierarchies be arranged in descending order?
A. I, II, III C. III, I, II
B. II, III, I D. III, II, I

393. Which statement best describes “value in use”?


A. The amount of cash that could currently be obtained by selling an asset in an orderly disposal.
B. The amount which an entity expects to obtain for an asset at the end of the useful life.
C. The present value of estimated future ash flows expected to arise from the continuing use of an
asset and from the ultimate disposal.
D. Undiscounted future net cash flows.

394. What is the best evidence of fair value?


A. Quoted price in an active market for similar asset
B. Quoted price in an active market for identical asset
C. Quoted price in an inactive market for similar asset
D. Quoted price in an inactive market for identical asset

395. If the fair value less cost to sell cannot be determined,


A. The asset is not impaired. C. The recoverable amount is the value in use.
B. The net realizable value is used. D. The carrying amount of the asset remains the same.

396. What is the allocation of an impairment loss recognized for a cash generating unit?
A. First to any goodwill, and the balance to the other assets prorata based on fair value.
B. First to any goodwill, and the balance to the other assets prorata based on carrying amount.
C. Across the assets of the unit based on fair value.
D. Across the assets of the unit based on carrying amount.

397. All of the following statements are true with regard to impairment of assets, except
A. The impairment test compares the carrying amount with the lower of fair value less cost of
disposal and value in use.
B. If impairment indicators are present, the entity must conduct an impairment test
C. If the recoverable amount is lower than carrying amount an impairment loss is recognized.
D. If recoverable amount is higher than carrying amount, no impairment loss is recognized.

398. The impairment rules for long-lived assets apply to all of the following, except
A. Land C. Building currently used in business
B. Financial instrument D. Computer used to run a production process

399. The estimates of future cash flows in calculating value in use include all of the following, except
A. Income tax payment
B. Cash inflows from the continuing use of the asset
C. Cash outflows incurred to generate the cash inflows from the continuing use of the asset
D. Net cash flows from the disposal of the asset at the end of the useful life

400. What is cash generating unit?


A. The smallest business segment
B. Any group of assets that generate cash flows
C. Any group of assets reported separately to management
D. The smallest group of assets that generate independent cash flows from continuing use

401. Which statement best describes investment property?


A. Property held for sale in the ordinary course of the business
B. Property held for use in the production and supply of goods or services and property held for
administrative purposes
C. Property held to earn rentals or for capital appreciation
D. Property held for capital appreciation
402. Which statement is true of the property is partly investment and partly owner-occupied?
I. If the investment and owner-occupied portions could not be sold or leased out separately,
the property is investment property if only an insignificant portion is held for manufacturing
or administrative purposes.
II. If the investment and owner-occupied portions could be sold or leased out separately, the
portions shall be accounted for separately as investment property and owner-occupied
property.
A. I only B. II only C. Both I and II D. Neither I nor II

403. Which statement is incorrect in determining the fair value of an investment property?
A. The fair value of investment property excludes prepaid or accrued operating lease income.
B. The fair value of investment property shall reflect market conditions at the end of the reporting
period.
C. If an office is leased on a furnished basis, the fair value of the office generally includes the fair
value of the furniture because the rental income relates to the furnished office.
D. An entity shall determine the fair value of investment property by deducting transaction cost
that may be incurred upon disposal.

404. If the entity uses fair value model for the investment property, which of the following statements are
true?
I. Changes in fair value are reported in profit or loss in the current period.
II. Changes in fair value are reported as an extraordinary gain
III. Changes in fair value are reported in other comprehensive income for the period.
IV. Changes in fair value are reported as deferred revenue for the period
V. The entity does not record depreciation on the investment property
VI. The entity should have value the property at cost less accumulated depreciation and
impairment.
VII. The entity should report the increase in fair value in other comprehensive income for the
period.
VIII. The entity depreciated the equipment using the normal depreciation method.
A. I and V B. II and VI C. III and VII D. IV and VIII

405. Transfers from investment property to property, plant and equipment are appropriate
A. The entity can never transfer property into another classification once it is classified as investment
property.
B. Only when the entity adopts the fair value model.
C. Based on the discretion of management.
D. When there is a change of use.

406. When the entity uses the cost model, transfer between investment property, owner-occupied
property and inventory shall be accounted for at
A. Fair value B. Carrying amount C. Cost D. Assessed value

407. If an inventory is transferred to investment property that is to be carried at fair value, the
remeasurement to fair value is
A. Included in equity C. Included in profit or loss
B. Included in retained earnings D. Accounted for as revaluation of inventory

408. An investment property shall be measured initially at


A. Cost C. Fair value less impairment
B. Cost less impairment D. Depreciable amount less impairment

409. An investment property is derecognized when


I. It is disposed to a third party.
II. It is permanently withdrawn from use.
III. No future economic benefits are expected from the disposal
A. I only B. I and II only C. I and III only D. I, II and III

410. Which of the following additional disclosures must be made when an entity chooses the cost model
as the accounting policy for investment property?
A. The fair value of the property C. The value in use of the property
B. The present value of the property D. The net realizable value of the property
411. An intangible asset shall be recognized if
A. It is probable that future economic benefits attributable to the asset will flow to the entity.
B. It is probable that future economic benefits attributable to the asset will flow to the entity and
the cost of the intangible asset can be measured reliably.
C. It is possible that future economic benefits attributable to the asset will flow to the entity and the
cost of the intangible asset can be measured reliably.
D. The cost of the intangible asset can be measured reliably.

412. Directly attributable costs of preparing the intangible asset for the intended use include all of the
following, except
A. Initial operating losses
B. Cost of testing whether the asset is functioning properly
C. Professional fees arising directly from bringing the asset to the working condition
D. Cost of employee benefit arising directly from brining the asset to the working condition

413. Which statement is false in relation to internally generated intangible asset?


A. The cost of an internally generated asset includes expenditure on training staff to operate the
asset.
B. Internally generated brand, masthead, publishing title, and customer list shall not be recognized
as an intangible asset.
C. The cost of internally generated asset comprises all directly attributable costs necessary to
produce and prepare the asset for the intended use.
D. Internally generated goodwill shall not be recognized as an intangible asset.

414. After initial recognition, an intangible asset shall be measured using


A. Cost model C. Cost model or revaluation model
B. Revaluation model D. Cost model or fair value model

415. An intangible asset is regarded as having an indefinite useful life when


A. The useful life of the intangible asset arises from contractual right.
B. The useful life of the asset arises from legal right.
C. There is no foreseeable limit to the period over which the asset is expected to generate net cash
inflows to the entity.
D. There is a foreseeable limit to the period over which the asset is expected to generate net cash
inflows to the entity.

416. What is the method of amortizing intangible asset?


A. The straight line method in all circumstances
B. A subjective amount of periodic amortization
C. The double declining balance in all circumstances
D. The straight line method, unless the pattern of the economic benefits can be determined
reliably

417. Factors in determining the useful life of an intangible asset include all, except
A. The amortization method
B. The expected use of the asset
C. Any legal or contractual provision
D. Any provision for renewal or extension of the legal life

418. Which of the following is not one of the criteria which must be met before development costs can be
capitalized?
A. The entity has sufficient financial resources to complete the project.
B. The entity can reliably identify the research costs incurred to bring the project to economic
feasibility.
C. The entity intends to complete the project and either use or sell the intangible asset.
D. The project has achieved technical feasibility.

419. Which is not considered a research and development activity?


A. Laboratory research aimed at discovery of new knowledge
B. Conceptual formulation and design of possible product or process
C. Routine on-going effort to refine, enrich or improve quality of existing product
D. Design, construction and operation of a pilot plant
420. Which of the following research and development costs should be capitalized and amortized over
current and future periods?
A. Labor and material costs incurred in building a prototype model
B. Administrative salaries allocated to research and development
C. Research findings purchased from another entity to aid a particular research project currently in
process
D. Cost of testing equipment that will also be used in another separate research and development
project scheduled to begin next year

421. Biological assets


A. Are found only in Biotech entities
B. Are living animals or living plants and must disclosed as a separate line item in the statement
of financial position
C. Must be measured at cost
D. Do not generally have future economic benefit

422. Agricultural activity includes all of the following, except


A. Ocean fishing C. Raising livestock
B. Aquaculture D. Perennial cropping

423. Agricultural produce as it grows on bearer plant is measured at the end of each reporting period prior
to harvest at
A. Fair value C. Fair value plus cost of disposal
B. Fair value less cost of disposal D. Fair value less cost of disposal at the point of harvest

424. A living plant with dual use is classified as


A. Bearer plant C. Biological asset
B. Investment property D. Inventory

425. Mature bearer plant is measured using


A. Cost model C. Either cost model or revaluation model
B. Revaluation model D. Either cost model or fair value model

426. Animals related to recreational activities are accounted for under what standard?
A. IAS 41 – Agriculture C. IAS 40 – Investment property
B. IAS 16 – Property, plant and equipment D. Either IAS 41 or IAS 16

427. Which of the following is unlikely to be used in fair value measurement of biological asset?
A. Quoted market price
B. The most recent market transaction price
C. The present value of the expected net cash flows
D. External independent valuation

428. Which of the following information shall be disclosed in relation to biological asset and agricultural
produce?
A. The aggregate gain or loss arising on the initial recognition of biological asset and agricultural
produce and from the change in fair value less cost of disposal of biological asset.
B. The total gain or loss from biological asset, agricultural produce, and from changes in fair value
less cost of disposal of biological asset.
C. Separate disclosure of the gain or loss relating to biological asset and agricultural produce.
D. There is no requirement in the standard to disclose separately any gain or loss.

429. Land that is related to agricultural activity is measure


A. At fair value.
B. In accordance with PAS 16, Property, Plant and Equipment, or PAS 40, Investment Property.
C. At fair value in combination with the biological asset.
D. At resale value separate from the biological asset.

430. All of the following are classified as agricultural produce, except


A. Sugar B. Wool C. Cotton D. Milk

You might also like